Acessibilidade / Reportar erro

Sob pressão: como líderes reagem a ameaças à identidade relacionada a sua liderança paradoxal

Bajo presión: cómo reaccionan los líderes a las amenazas de identidad relacionadas con su liderazgo paradójico

Resumo

O objetivo deste estudo foi compreender como indivíduos com uma identidade de liderança paradoxal respondem a ameaças a essa identidade. Para isso, foi realizado um estudo de natureza qualitativa, por meio de entrevistas com 44 líderes de uma cooperativa de crédito brasileira. Após a realização de uma análise de conteúdo, identificamos cinco respostas de enfrentamento, que codificamos como “abandonando a identidade”, “defendendo a identidade”, “desacreditando ameaçadores”, “alterando o significado da identidade” e “relativizando a relevância da ameaça”. O estudo conclui que a escolha do tipo de estratégia de enfrentamento é crucial para a preservação ou reformulação da identidade de liderança paradoxal de um indivíduo.

Palavras-chave:
Liderança paradoxal; Ameaça à identidade; Respostas de enfrentamento

Resumen

El objetivo de este estudio fue comprender cómo los individuos con una identidad de liderazgo paradójico responden a las amenazas a esa identidad. Para ello, se realizó un estudio cualitativo, a través de entrevistas con 44 líderes de una cooperativa de crédito brasileña. Después de realizar un análisis de contenido, identificamos cinco respuestas de afrontamiento, que codificamos como “Abandono de la identidad”, “Defensa de la identidad”, Desacreditación de las amenazas”, “Cambio del significado de la identidad” y “Relativización de la relevancia de la amenaza”. El estudio concluye que la elección del tipo de estrategia de afrontamiento es crucial para preservar o reformular la identidad de liderazgo paradójico de un individuo.

Palabras clave:
Liderazgo paradójico; Amenaza de identidad; Respuestas de afrontamiento

Abstract

The aim of this study was to understand how individuals with a paradoxical leadership identity respond to threats to that identity. To this end, we conducted a qualitative study, through interviews with 44 leaders of a Brazilian credit cooperative. After performing a content analysis, we identified five coping responses, which we coded as “Abandoning the identity”, “Defending the identity”, Discrediting threatenings”, “Changing the meaning of the identity” and “Relativizing the relevance of the threat”. The study concludes that the choice of the type of coping strategy is crucial for the preservation or reformulation of the individual’s paradoxical leadership identity.

Keywords:
Paradoxical leadership; Identity threat; Coping responses

INTRODUÇÃO

Em um mercado de trabalho cada vez mais competitivo, organizações têm buscado contratar e formar líderes capazes de gerenciar demandas contraditórias (W. K. Smith & Lewis, 2011Smith, W. K., & Lewis, M. W. (2011). Toward a theory of paradox: a dynamic equilibrium model of organizing. Academy of Management Review, 36(2), 381-403. Recuperado de https://doi.org/10.5465/amr.2009.0223
https://doi.org/10.5465/amr.2009.0223...
) e suas tensões resultantes (Cavalcanti, Felix, & Mainardes, 2022Cavalcanti, A. L., Felix, B., & Mainardes, E. W. (2022). Do tensions lead to positive career satisfaction results? Revista de Administração Mackenzie, 23(3), 1-26. Recuperado de https://doi.org/10.1590/1678-6971/eRAMR220200.en
https://doi.org/10.1590/1678-6971/eRAMR2...
). Espera-se, por exemplo, que líderes sejam capazes de enfatizar valores organizacionais sociais, ao mesmo tempo que incentivam o alcance de metas financeiras (W. K. Smith, Besharov, Wessels, & Chertok, 2012Smith, W. K., & Lewis, M. W. (2011). Toward a theory of paradox: a dynamic equilibrium model of organizing. Academy of Management Review, 36(2), 381-403. Recuperado de https://doi.org/10.5465/amr.2009.0223
https://doi.org/10.5465/amr.2009.0223...
), de promover a implementação de padrões simultaneamente à adoção de práticas de inovação (Shao, Nijstad, & Täuber, 2019Shao, Y., Nijstad, B. A., & Täuber, S. (2019). Creativity under workload pressure and integrative complexity: The double-edged sword of paradoxical leadership. Organizational Behavior and Human Decision Processes, 155, 7-19. Recuperado de https://doi.org/10.1016/j.obhdp.2019.01.008
https://doi.org/10.1016/j.obhdp.2019.01....
) e de liderar com base em regras coletivas, sem deixar de considerar necessidades individuais (Volk, Waldman, & Barnes, 2022Volk, S., Waldman, D. A., & Barnes, C. M. (2022). A circadian theory of paradoxical leadership. Academy of Management Review, 48(4), 611-638. Recuperado de https://doi.org/10.5465/amr.2020.0468
https://doi.org/10.5465/amr.2020.0468...
). Assim, gerenciar essas demandas paradoxais tem sido cada vez mais relevante para indivíduos em posição de liderança (Sousa et al., 2022Sousa, M., Cunha, M. P. E., Simpson, A. V., Giustiniano, L., Rego, A., & Clegg, S. (2022). Servus or Pater? How Paradoxical Intent can qualify Leadership: Inductions from the Kingdom of Bhutan. Journal of Change Management: Reframing Leadership and Organizational Practice, 22(3), 321-353. Recuperado de https://doi.org/10.1080/14697017.2022.2032271
https://doi.org/10.1080/14697017.2022.20...
).

No entanto, nem sempre esta foi uma visão popular nos estudos de liderança. Tradicionalmente, teorias de liderança situacional ou contingencial (p. ex., Howell, Dorfman, & Kerr, 1986Howell, J. P., Dorfman, P. W., & Kerr, S. (1986). Moderator variables in leadership research. Academy of Management Review, 11(1), 88-102. Recuperado de https://doi.org/10.5465/amr.1986.4282632
https://doi.org/10.5465/amr.1986.4282632...
) adotaram uma perspectiva de dilema para lidar com as citadas tensões. Nessa abordagem, A e B são vistos como polaridades independentes e contraditórias, que devem ser adotadas em condições específicas (Luthans & Stewart, 1977Luthans, F., & Stewart, T. I. (1977). A General Contingency Theory of Management. Academy of Management Review, 2(2), 181-195. Recuperado de https://doi.org/10.5465/amr.1977.4409038
https://doi.org/10.5465/amr.1977.4409038...
). Assim, escolher entre tensões mutuamente excludentes (A ou B) era visto como um mal necessário (Zhang, Waldman, Han, & Li, 2015Zhang, Y., Waldman, D. A., Han, Y. L., & Li, X. B. (2015). Paradoxical leader behavior in people management: antecedents and consequences. Academy of Management Journal, 58(2), 538-566. Recuperado de https://doi.org/10.5465/amj.2012.0995
https://doi.org/10.5465/amj.2012.0995...
). Nesta pesquisa, entretanto, explora-se uma visão emergente e contrastante de como líderes devem lidar com demandas contraditórias. Exploramos o conceito de liderança paradoxal, que é a “[...] integração entre comportamentos de liderança que são aparentemente contraditórios, mas também interdependentes” (Volk et al., 2022Volk, S., Waldman, D. A., & Barnes, C. M. (2022). A circadian theory of paradoxical leadership. Academy of Management Review, 48(4), 611-638. Recuperado de https://doi.org/10.5465/amr.2020.0468
https://doi.org/10.5465/amr.2020.0468...
, p. 1). Nesse caso, não se realiza uma escolha entre A ou B, mas se optapor A e B simultaneamente.

Enquanto os desenvolvimentos teóricos sobre liderança paradoxal são relativamente recentes (Pearce, Wassenaar, Berson, & Tuval-Mashiach, 2019Pearce, C. L., Wassenaar, C. L., Berson, Y., & Tuval-Mashiach, R. (2019). Toward a theory of meta-paradoxical leadership. Organizational Behavior and Human Decision Processes, 155, 31-41. Recuperado de https://doi.org/10.1016/j.obhdp.2019.03.003
https://doi.org/10.1016/j.obhdp.2019.03....
; W. K. Smith et al., 2012Smith, W. K., Besharov, M. L., Wessels, A. K., & Chertok, M. (2012). A paradoxical leadership model for social entrepreneurs: challenges, leadership skills, and pedagogical tools for managing social and commercial demands. Academy of Management Learning and Education, 11(3), 463-478. Recuperado de https://doi.org/10.5465/amle.2011.0021
https://doi.org/10.5465/amle.2011.0021...
; Volk et al., 2022Volk, S., Waldman, D. A., & Barnes, C. M. (2022). A circadian theory of paradoxical leadership. Academy of Management Review, 48(4), 611-638. Recuperado de https://doi.org/10.5465/amr.2020.0468
https://doi.org/10.5465/amr.2020.0468...
; Zhang et al., 2015Zhang, Y., Waldman, D. A., Han, Y. L., & Li, X. B. (2015). Paradoxical leader behavior in people management: antecedents and consequences. Academy of Management Journal, 58(2), 538-566. Recuperado de https://doi.org/10.5465/amj.2012.0995
https://doi.org/10.5465/amj.2012.0995...
), no nível organizacional a ideia de paradoxos tem sido discutida há mais tempo (Gibson & Birkinshaw, 2004Gibson, C. B., & Birkinshaw, J. (2004). The antecedents, consequences, and mediating role of organizational ambidexterity. Academy of Management Journal, 47(2), 209-226. Recuperado de https://doi.org/10.5465/20159573
https://doi.org/10.5465/20159573...
; O’Reilly & Tushman, 2008O’Reilly, C. A., & Tushman, M. L. (2008). Ambidexterity as a dynamic capability: Resolving the innovator’s dilemma. Research in Organizational Behavior, 28, 185-206. Recuperado de https://doi.org/10.1016/j.riob.2008.06.002
https://doi.org/10.1016/j.riob.2008.06.0...
; Raisch & Birkinshaw, 2008Raisch, S., & Birkinshaw, J. (2008). Organizational ambidexterity: antecedents, outcomes, and moderators. Journal of Management, 34(3), 375-409. Recuperado de https://doi.org/10.1177/0149206308316058
https://doi.org/10.1177/0149206308316058...
; W. K. Smith et al., 2012Smith, W. K., & Lewis, M. W. (2011). Toward a theory of paradox: a dynamic equilibrium model of organizing. Academy of Management Review, 36(2), 381-403. Recuperado de https://doi.org/10.5465/amr.2009.0223
https://doi.org/10.5465/amr.2009.0223...
). Com isso, avanços já identificados na literatura de paradoxos organizacionais precisam ser aprofundados também no nível individual de análise. Isso é relevante porque a ideia de paradoxos não permeia apenas a construção de estruturas meso e estratégias corporativas (W. K. Smith et al., 2012Smith, W. K., & Lewis, M. W. (2011). Toward a theory of paradox: a dynamic equilibrium model of organizing. Academy of Management Review, 36(2), 381-403. Recuperado de https://doi.org/10.5465/amr.2009.0223
https://doi.org/10.5465/amr.2009.0223...
), mas também ações de nível individual (Miron-Spektor, Ingram, Keller, W. K. Smith, & Lewis, 2018Miron-Spektor, E., Ingram, A., Keller, J., Smith, W. K., & Lewis, M. W. (2018). Microfoundations of organizational paradox: the problem is how we think about the problem. Academy of Management Journal, 61(1), 26-45. Recuperado de https://doi.org/10.5465/amj.2016.0594
https://doi.org/10.5465/amj.2016.0594...
), como é o caso de líderes (Sulphey & Jasim, 2022Sulphey, M. M., & Jasim, K. M. (2022). Paradoxical leadership as a moderating factor in the relationship between organizational silence and employee voice: an examination using SEM. Leadership & Organization Development Journal, 43(3), 457-481. Recuperado de https://doi.org/10.1108/LODJ-02-2021-0075
https://doi.org/10.1108/LODJ-02-2021-007...
). Um desses avanços obtidos no nível organizacional e que precisa ser aprofundado é o das implicações que paradoxos trazem para a ideia de identidade.

O trabalho seminal de Albert e Whetten (1985Albert, S., & Whetten, D. A. (1985). Organizational identity. In L. L. Cummings & B.M. Staw (Eds.), Research in Organizational Behavior (pp. 263-295). London, UK: Jay Press.) propôs que a identidade de uma organização deveria enfatizar um traço dominante (centralidade). Entretanto, essa visão monolítica das identidades organizacionais tem sido contestada. Alguns acadêmicos têm sugerido que, por serem sistemas abertos, diversas organizações têm enfrentado pressões paradoxais e respondido a elas com identidades organizacionais mais híbridas e pluralistas, o que desafia a tradicional ideia de um único traço dominante para a identidade de uma organização (Ashforth & Reingen, 2014Ashforth, B. E., & Reingen, P. H. (2014). Functions of dysfunction: managing the dynamics of an organizational duality in a natural food cooperative. Administrative Science Quarterly, 59(3), 474-516. Recuperado de https://doi.org/10.1177/0001839214537811
https://doi.org/10.1177/0001839214537811...
; Fiol, 2002Fiol, C. M. (2002). Capitalizing on paradox: the role of language in transforming organizational identities. Organization Science, 13(6), 653-666. Recuperado de https://doi.org/10.1287/orsc.13.6.653.502
https://doi.org/10.1287/orsc.13.6.653.50...
; Jäger & Schröer, 2014Jäger, U. P., & Schröer, A. (2014). Integrated organizational identity: a definition of hybrid organizations and a research agenda. Voluntas, 25(5), 1281-1306. Recuperado de https://doi.org/10.1007/s11266-013-9386-1
https://doi.org/10.1007/s11266-013-9386-...
; Kozica, Gebhardt, Müller-Seitz, & Kaiser, 2015Kozica, A. M. F., Gebhardt, C., Müller-Seitz, G., & Kaiser, S. (2015). Organizational identity and paradox: an analysis of the “stable state of instability” of Wikipedia’s identity. Journal of Management Inquiry, 24(2), 186-203. Recuperado de https://doi.org/10.1177/1056492614553275
https://doi.org/10.1177/1056492614553275...
; Kreiner, Hollensbe, Sheep, B. R. Smith, & Kataria, 2015Kreiner, G. E., Hollensbe, E., Sheep, M. L., Smith, B. R., & Kataria, N. (2015). Elasticity and the dialectic tensions of organizational identity: how can we hold together while we’re pulling apart? Academy of Management Journal, 58(4), 981-1011. Recuperado de https://doi.org/10.5465/amj.2012.0462
https://doi.org/10.5465/amj.2012.0462...
). Assim, tem crescido a percepção, para algumas organizações, de formação de uma identidade organizacional paradoxal, na qual, em vez de um traço único central, há uma polaridade central que constitui o “quem nós somos” de uma instituição. Felix (2020aFelix, B. (2020a). Analyzing the formation of a paradoxical organizational identity. International Journal of Organizational Analysis, 28(6), 1227-1241. Recuperado de https://doi.org/10.1108/IJOA-08-2019-1849
https://doi.org/10.1108/IJOA-08-2019-184...
, 2021Felix, B. (2020b). The (un)speaking self: an identity-based model for employee voice and silence. Cadernos EBAPE.BR, 18(3), 557-571. Recuperado de https://doi.org/10.1590/1679-395120190037
https://doi.org/10.1590/1679-39512019003...
), por exemplo, aponta a formação de identidades organizacionais paradoxais em cooperativas de créditos, nas quais o traço de polaridade central identificado foi a dinâmica entre idealismo (cooperativismo) e pragmatismo (resultados financeiros). Ambos os trabalhos indicaram que, para diversos membros das organizações estudadas, o processo de construção social de uma identidade organizacional paradoxal mostrou-se complexo, uma vez que muitos tendiam a interpretar a organização com base em dilemas e, não, em paradoxos. Em outras palavras, tinham propensão a interpretar as organizações estudadas ou como idealistas ou como pragmatistas.

No nível individual de análise, a literatura sobre identidade da liderança sugere que ela não é fruto apenas de cognições que residem no autoconceito do líder (Day & Harrison, 2007Day, D. v., & Harrison, M. M. (2007). A multilevel, identity-based approach to leadership development. Human Resource Management Review, 17(4), 360-373. Recuperado de https://doi.org/10.1016/j.hrmr.2007.08.007
https://doi.org/10.1016/j.hrmr.2007.08.0...
; DeRue, Ashford, & Cotton, 2009DeRue, D. S., Ashford, S. J., & Cotton, N. C. (2009). Assuming the mantle: unpacking the process by which individuals internalize a leader identity. In L. M. Roberts & J. E. Dutton (Eds.), Exploring positive identities and organizations: Building a theoretical and research foundation (pp. 213-232). London, UK: Psychology Press.). Ela é também construída com base em interações sociais e, após o período inicial de aquisição de uma nova identidade (Ashforth, 2000Ashforth, B. E. (2000). Role transitions in organizational life: an identity-based perspective. New York, NY: Routledge.), depende de validação por parte dos seguidores (DeRue & Ashford, 2010DeRue, D. S., Ashford, S. J., & Cotton, N. C. (2009). Assuming the mantle: unpacking the process by which individuals internalize a leader identity. In L. M. Roberts & J. E. Dutton (Eds.), Exploring positive identities and organizations: Building a theoretical and research foundation (pp. 213-232). London, UK: Psychology Press.). Assim, entendemos, neste trabalho, que um indivíduo que apresenta comportamentos de liderança paradoxal constrói um autoconceito fundamentado em uma polaridade central (p. ex., ênfase simultânea em regras coletivas e necessidades individuais de seus seguidores) e tem esse autoconceito validado por seus seguidores. Pode-se dizer que ele apresenta uma identidade de liderança paradoxal. Dadas as recentes evidências das consequências positivas da liderança paradoxal (p. ex., Sparr, Knippenberg, & Kearney, 2022Sparr, J. L., Knippenberg, D. van, & Kearney, E. (2022). Paradoxical leadership as sensegiving: stimulating change-readiness and change-oriented performance. Leadership & Organization Development Journal, 43(2), 225-237. Recuperado de https://doi.org/10.1108/LODJ-04-2021-0161
https://doi.org/10.1108/LODJ-04-2021-016...
; Sulphey & Jasim, 2022Sulphey, M. M., & Jasim, K. M. (2022). Paradoxical leadership as a moderating factor in the relationship between organizational silence and employee voice: an examination using SEM. Leadership & Organization Development Journal, 43(3), 457-481. Recuperado de https://doi.org/10.1108/LODJ-02-2021-0075
https://doi.org/10.1108/LODJ-02-2021-007...
; Zhang, Zhang, Law, & Zhou, 2022Zhang, Y., Zhang, Y., Law, K. S., & Zhou, J. (2022). Paradoxical leadership, subjective ambivalence, and employee creativity: effects of employee holistic thinking. Journal of Management Studies, 59(3), 695-723. Recuperado de https://doi.org/10.1111/joms.12792
https://doi.org/10.1111/joms.12792...
) e o fato de que a identidade de um indivíduo cumpre a função de lhe prover autoestima e eficácia social (Felix, 2020bFelix, B. (2020b). The (un)speaking self: an identity-based model for employee voice and silence. Cadernos EBAPE.BR, 18(3), 557-571. Recuperado de https://doi.org/10.1590/1679-395120190037
https://doi.org/10.1590/1679-39512019003...
; Leavitt & Sluss, 2015Leavitt, K., & Sluss, D. M. (2015). Lying for who we are: an identity-based model of workplace dishonesty. Academy of Management Review, 40(4), 587-610. Recuperado de https://doi.org/10.5465/amr.2013.0167
https://doi.org/10.5465/amr.2013.0167...
), nota-se a relevância de melhor compreender as dinâmicas sociais pelas quais indivíduos vivenciam sua identidade de liderança paradoxal.

Dentre essas, uma dinâmica interacional, em especial, nos interessa neste trabalho: uma vez que indivíduos frequentemente interpretam a realidade com base em dilemas (A ou B) e não em paradoxos (A e B), é provável que o autoconceito paradoxal de alguns líderes seja colocado em xeque em algumas interações sociais com seus liderados (Felix, 2020aFelix, B. (2020a). Analyzing the formation of a paradoxical organizational identity. International Journal of Organizational Analysis, 28(6), 1227-1241. Recuperado de https://doi.org/10.1108/IJOA-08-2019-1849
https://doi.org/10.1108/IJOA-08-2019-184...
). Por exemplo, alguns seguidores os quais esperam que o líder seja “pragmatista” ou “idealista” podem ter dificuldades de seguir e validar socialmente sua identidade “pragmatista e idealista” (Felix, 2021Felix, B., Dourado, D., & Nossa, V. (2023). Algorithmic management, preferences for autonomy/security and gig-workers’ wellbeing: A matter of fit? Frontiers in Psychology, 14, 1-12. Recuperado de https://doi.org/10.3389/fpsyg.2023.1088183
https://doi.org/10.3389/fpsyg.2023.10881...
). Dependendo da susceptibilidade do líder à falta de validação de seus seguidores, ele pode vivenciar o que a literatura chama de ameaça à identidade (Petriglieri, 2011Petriglieri, J. L. (2011). Under threat: responses to and the consequences of threats to individuals’ identities. Academy of Management Review, 36(4), 641-662. Recuperado de https://doi.org/10.5465/amr.2009.0087
https://doi.org/10.5465/amr.2009.0087...
), que é definida como uma situação na qual um indivíduo percebe que seu contexto social o impede de afirmar um aspecto de sua identidade (Elsbach, 2003Elsbach, K. D. (2003). Relating physical environment to self-categorizations: identity threat and affirmation in a non-territorial office space. Administrative Science Quarterly, 48(4), 622-654. Recuperado de https://doi.org/10.2307/3556639
https://doi.org/10.2307/3556639...
). Estudos mostram que ameaças à identidade podem levar à queda de performance e autoestima (Steele, 1997.Steele, C. M(1997). A threat in the air: how stereotypes shape intellectual identity and performance. American Psychologist, 52(6), 613-629. Recuperado de https://doi.org/10.1037/0003-066X.52.6.613
https://doi.org/10.1037/0003-066X.52.6.6...
), a um aumento nas chances de pedidos de demissão (Trevor & Nyberg, 2008Trevor, C. O., & Nyberg, A. J. (2008). Keeping your headcount when all about you are losing theirs: downsizing, voluntary turnover rates, and the moderating role of HR practices. Academy of Management Journal, 51(2), 259-276. Recuperado de https://doi.org/10.5465/amj.2008.31767250
https://doi.org/10.5465/amj.2008.3176725...
) e a uma menor tendência a aceitar cargos de liderança (Davies, Spencer, & Steele, 2005Davies, P. G., Spencer, S. J., & Steele, C. M., & (2005). Clearing the air: Identity safety moderates the effects of stereotype threat on women’s leadership aspirations. Journal of Personality and Social Psychology, 88(2), 276-287. Recuperado de https://doi.org/10.1037/0022-3514.88.2.276
https://doi.org/10.1037/0022-3514.88.2.2...
). Apesar da relevância do tema, ainda se faz necessário melhor compreender o fenômeno da ameaça à identidade de líderes paradoxais, suas respostas a essas ameaças e as implicações de ambos para organizações.

Para realizar esse avanço na literatura, objetivamos com este estudo compreender como indivíduos com uma identidade de liderança paradoxal respondem a ameaças a essa identidade. Para isso, foi desenvolvida uma pesquisa de natureza qualitativa baseada em entrevistas com funcionários de uma cooperativa de crédito brasileira. Esta pesquisa contribui para a teoria e prática sobre liderança paradoxal ao usar uma lente de identidade que permita compreender como indivíduos sustentam e/ou modificam aspectos de seus autoconceitos, os quais envolvem a liderança paradoxal. A despeito de a literatura prévia haver discutido questões de identidade paradoxal organizacional (Felix, 2020aFelix, B. (2020a). Analyzing the formation of a paradoxical organizational identity. International Journal of Organizational Analysis, 28(6), 1227-1241. Recuperado de https://doi.org/10.1108/IJOA-08-2019-1849
https://doi.org/10.1108/IJOA-08-2019-184...
), este trabalho expande essa discussão para o nível individual de análise, mais especificamente para a ideia de identidade de liderança. Assim, contribuímos para a literatura ao teorizar sobre conceitos relevantes (ameaça à identidade de liderança paradoxal e as respostas a elas), explicar como tais conceitos se relacionam (como indivíduos oferecem diferentes respostas conforme a vivência da ameaça) e tratar da importância do fenômeno (porque isso importa).

REVISÃO DA LITERATURA

Liderança Paradoxal

Um paradoxo refere-se à existência de duas polaridades que são desejáveis e guardam entre si uma relação de tensão e conflito, ainda que sejam interdependentes (W. K. Smith & Lewis, 2011Smith, W. K., & Lewis, M. W. (2011). Toward a theory of paradox: a dynamic equilibrium model of organizing. Academy of Management Review, 36(2), 381-403. Recuperado de https://doi.org/10.5465/amr.2009.0223
https://doi.org/10.5465/amr.2009.0223...
). Esse conceito, tradicional nos campos da Filosofia e Psicologia (Miron-Spektor et al., 2018Miron-Spektor, E., Ingram, A., Keller, J., Smith, W. K., & Lewis, M. W. (2018). Microfoundations of organizational paradox: the problem is how we think about the problem. Academy of Management Journal, 61(1), 26-45. Recuperado de https://doi.org/10.5465/amj.2016.0594
https://doi.org/10.5465/amj.2016.0594...
), tem sido explorado mais recentemente no contexto de organizações do trabalho (Lüscher & Lewis, 2008Lüscher, L. S., & Lewis, M. W. (2008). Organizational change and managerial sensemaking: working through paradox. Academy of Management Journal, 51(2), 221-240. Recuperado de https://doi.org/10.5465/amj.2008.31767217
https://doi.org/10.5465/amj.2008.3176721...
; W. K. Smith et al., 2012Smith, W. K., & Lewis, M. W. (2011). Toward a theory of paradox: a dynamic equilibrium model of organizing. Academy of Management Review, 36(2), 381-403. Recuperado de https://doi.org/10.5465/amr.2009.0223
https://doi.org/10.5465/amr.2009.0223...
). Diversos acadêmicos, de Laozi a Kierkegaard e Jung, teorizaram sobre a faca de dois gumes contida em elementos contraditórios da vida humana (Lewis, 2000Lewis, M. 2000. Exploring paradox: Toward a more comprehensive guide. Academy of Management Review, 25(4), 760-776. Recuperado de https://doi.org/10.5465/amr.2000.3707712
https://doi.org/10.5465/amr.2000.3707712...
). Por um lado, estudos tradicionais mostraram que tensões entre demandas contraditórias, se tratadas como interdependentes, podem levar a resultados positivos. Um exemplo é o estudo de Eisenhardt e Westcott (1988Eisenhardt, K. M., & Westcott, B. J. (1988). Paradoxical demands and the creation of excellence: The case of just-in-time manufacturing. In R. Quinn & K. Cameron (Eds.), Paradox and transformation: Toward a theory of change in organization and management (pp. 19-54). Cambridge, MA: Ballinger.), que mostrou como líderes da Toyota adotavam uma perspectiva de paradoxos ao encarar demandas conflituosas entre si, como prazo e qualidade, não como excludentes, mas como sinérgicas. Já Rothenberg (1979Rothenberg, A. (1979). The emerging goddess. Chicago: University of Chicago Press.) ressaltou como expoentes da arte e ciência, tais como Mozart, Einstein e Picasso, utilizaram a estratégia de conciliar a pressão por produzir com o ócio reflexivo para criar suas obras. Por outro lado, há também quem mostre que as tensões entre essas polaridades e demandas contraditórias podem ameaçar nossa necessidade por estabilidade cognitiva e emocional, desafiar nossos egos e provocar respostas defensivas que levam a uma paralisia diante de conflitos vistos como irreconciliáveis (K. Smith & Berg, 1987Waldman, D. A., & Bowen, D. E. (2016). Learning to be a paradox-savvy leader. Academy of Management Perspectives, 30(3), 316-327. Recuperado de https://doi.org/10.5465/amp.2015.0070
https://doi.org/10.5465/amp.2015.0070...
; Vince & Broussine, 1996Vince, R., & Broussine, M. (1996). Paradox, defense and attachment: Accessing and working with emotions and relations underlying organizational change. Organization Studies, 17(1) 1-21. Recuperado de https://doi.org/10.1177/017084069601700101
https://doi.org/10.1177/0170840696017001...
).

Mais especificamente, a emergência do conceito de paradoxos organizacionais tem levado a uma crescente busca por compreender como líderes lidam com pressões, tensões e demandas contraditórias e interdependentes (Lüscher & Lewis, 2008; W. K. Smith & Lewis, 2011Smith, W. K., & Lewis, M. W. (2011). Toward a theory of paradox: a dynamic equilibrium model of organizing. Academy of Management Review, 36(2), 381-403. Recuperado de https://doi.org/10.5465/amr.2009.0223
https://doi.org/10.5465/amr.2009.0223...
) e o que leva alguns gestores a alcançar sucesso nessa tarefa, enquanto outros não são exitosos (Fürstenberg, Alfes, & Kearney, 2021Fürstenberg, N., Alfes, K., & Kearney, E. (2021). How and when paradoxical leadership benefits work engagement: the role of goal clarity and work autonomy. Journal of Occupational and Organizational Psychology, 94(3), 672-705. Recuperado de https://doi.org/10.1111/joop.12344
https://doi.org/10.1111/joop.12344...
; Pan, 2021Pan, Z. (2021). Paradoxical leadership and organizational citizenship behavior: the serial mediating effect of a paradoxical mindset and personal service orientation. Leadership and Organization Development Journal, 42(6), 869-881. Recuperado de https://doi.org/10.1108/LODJ-08-2020-0351
https://doi.org/10.1108/LODJ-08-2020-035...
; She, Li, Yang, & Yang, 2020She, Z., Li, Q., Yang, B., & Yang, B. (2020). Paradoxical leadership and hospitality employees’ service performance: the role of leader identification and need for cognitive closure. International Journal of Hospitality Management, 89, (102524), 1-11. Recuperado de https://doi.org/10.1016/j.ijhm.2020.102524
https://doi.org/10.1016/j.ijhm.2020.1025...
; W. K. Smith et al., 2012Smith, W. K., & Lewis, M. W. (2011). Toward a theory of paradox: a dynamic equilibrium model of organizing. Academy of Management Review, 36(2), 381-403. Recuperado de https://doi.org/10.5465/amr.2009.0223
https://doi.org/10.5465/amr.2009.0223...
; Waldman & Bowen, 2016Waldman, D. A., & Bowen, D. E. (2016). Learning to be a paradox-savvy leader. Academy of Management Perspectives, 30(3), 316-327. Recuperado de https://doi.org/10.5465/amp.2015.0070
https://doi.org/10.5465/amp.2015.0070...
). Nesse contexto, surge o conceito de liderança paradoxal como uma abordagem de liderança que seria mais adequada para produzir resultados satisfatórios em contextos ambientais dinâmicos e complexos (Yi, Mao, & Wang, 2019Yi, L., Mao, H., & Wang, Z. (2019). How paradoxical leadership affects ambidextrous innovation: the role of knowledge sharing. Social Behavior and Personality: An International Journal, 47(4), 1-15. Recuperado de https://doi.org/10.2224/sbp.7636
https://doi.org/10.2224/sbp.7636...
; Zhang et al., 2015Zhang, Y., Waldman, D. A., Han, Y. L., & Li, X. B. (2015). Paradoxical leader behavior in people management: antecedents and consequences. Academy of Management Journal, 58(2), 538-566. Recuperado de https://doi.org/10.5465/amj.2012.0995
https://doi.org/10.5465/amj.2012.0995...
). O termo se refere à “[...] integração entre comportamentos de liderança que são aparentemente contraditórios, mas também interdependentes” (Volk et al., 2022Volk, S., Waldman, D. A., & Barnes, C. M. (2022). A circadian theory of paradoxical leadership. Academy of Management Review, 48(4), 611-638. Recuperado de https://doi.org/10.5465/amr.2020.0468
https://doi.org/10.5465/amr.2020.0468...
, p. 1). De acordo com a perspectiva de liderança paradoxal, o líder deve ser capaz de adotar, em resposta a pressões contingenciais complexas, comportamentos que apresentam uma certa contradição entre si, ainda que sejam interdependentes (Backhaus, Reuber, D. Vogel, & R. Vogel, 2021Backhaus, L., Reuber, A., Vogel, D., & Vogel, R. (2021). Giving sense about paradoxes: paradoxical leadership in the public sector. Public Management Review, 24(9), 1478-1498. Recuperado de https://doi.org/10.1080/14719037.2021.1906935
https://doi.org/10.1080/14719037.2021.19...
).

Temos um exemplo disso quando líderes se deparam com a necessidade de impulsionar a performance de sua equipe e, ao mesmo tempo, de promover o senso de pertencimento de seus seguidores (W. K. Smith & Lewis, 2011Smith, W. K., & Lewis, M. W. (2011). Toward a theory of paradox: a dynamic equilibrium model of organizing. Academy of Management Review, 36(2), 381-403. Recuperado de https://doi.org/10.5465/amr.2009.0223
https://doi.org/10.5465/amr.2009.0223...
). Um líder sem a mentalidade de paradoxo tenderia a pensar que, para alcançar a performance, provavelmente deveria sacrificar o nível do senso de pertencimento de seus seguidores em relação ao grupo. Outro comportamento recorrente de um líder que não adota a abordagem paradoxal seria cultivar o senso de pertencimento e conexão de seus seguidores à custa do desempenho, deixando de corrigir ou desafiar membros da equipe por receio da perda de senso de grupo. Em ambos os casos, o líder estaria adotando uma mentalidade de dilema (Miron-Spektor et al., 2018Miron-Spektor, E., Ingram, A., Keller, J., Smith, W. K., & Lewis, M. W. (2018). Microfoundations of organizational paradox: the problem is how we think about the problem. Academy of Management Journal, 61(1), 26-45. Recuperado de https://doi.org/10.5465/amj.2016.0594
https://doi.org/10.5465/amj.2016.0594...
; W. K. Smith et al., 2012Smith, W. K., & Lewis, M. W. (2011). Toward a theory of paradox: a dynamic equilibrium model of organizing. Academy of Management Review, 36(2), 381-403. Recuperado de https://doi.org/10.5465/amr.2009.0223
https://doi.org/10.5465/amr.2009.0223...
), segundo a qual se entende que uma polaridade pode ser alcançada apesar da outra. Já no caso da liderança paradoxal, a mentalidade é de que o senso de pertencimento dos seguidores deve ser estimulado via melhoria do desempenho e vice-versa. Nesse caso, a ideia é de que o senso de “nós” deve ser construído baseado na união para atingir metas ou na comemoração de conquistas. Assim, as polaridades do paradoxo (neste exemplo, performar e pertencer) são vistas como interdependentes (Zhang et al., 2015Zhang, Y., Waldman, D. A., Han, Y. L., & Li, X. B. (2015). Paradoxical leader behavior in people management: antecedents and consequences. Academy of Management Journal, 58(2), 538-566. Recuperado de https://doi.org/10.5465/amj.2012.0995
https://doi.org/10.5465/amj.2012.0995...
), e não como uma escolha a ser feita entre demandas mutuamente excludentes (Miron-Spektor et al., 2018Miron-Spektor, E., Ingram, A., Keller, J., Smith, W. K., & Lewis, M. W. (2018). Microfoundations of organizational paradox: the problem is how we think about the problem. Academy of Management Journal, 61(1), 26-45. Recuperado de https://doi.org/10.5465/amj.2016.0594
https://doi.org/10.5465/amj.2016.0594...
).

Líderes são submetidos a demandas paradoxais, mas eles não são os únicos. Tais pressões também atingem seus seguidores, motivo pelo qual se espera que estes também apresentem comportamentos paradoxais para que um melhor resultado seja obtido (Li, Xue, Liang, & Yan, 2020Xue, Y., Li, X., Liang, H., & Li, Y. (2020). How does paradoxical leadership affect employees’ voice behaviors in workplace? A leader-member exchange perspective. International Journal of Environmental Research and Public Health, 17(4), 1162-1185. Recuperado de https://doi.org/10.3390/ijerph17041162
https://doi.org/10.3390/ijerph17041162...
; Sparr et al., 2022Sparr, J. L., Knippenberg, D. van, & Kearney, E. (2022). Paradoxical leadership as sensegiving: stimulating change-readiness and change-oriented performance. Leadership & Organization Development Journal, 43(2), 225-237. Recuperado de https://doi.org/10.1108/LODJ-04-2021-0161
https://doi.org/10.1108/LODJ-04-2021-016...
). Em alguns casos, os seguidores se juntam aos seus líderes na tarefa de adotar comportamentos paradoxais (Felix, 2021Felix, B. (2021). Batalhando com espadas e escudos: uma análise semântica do paradoxo entre pertencimento e desempenho em uma cooperativa. Revista Organizações & Sociedade, 28(99), 786-805. Recuperado de https://doi.org/10.1590/1984-92302021v28n9903PT
https://doi.org/10.1590/1984-92302021v28...
; Miron-Spektor et al., 2018Miron-Spektor, E., Ingram, A., Keller, J., Smith, W. K., & Lewis, M. W. (2018). Microfoundations of organizational paradox: the problem is how we think about the problem. Academy of Management Journal, 61(1), 26-45. Recuperado de https://doi.org/10.5465/amj.2016.0594
https://doi.org/10.5465/amj.2016.0594...
). No entanto, em outros casos, a maneira como seguidores percebem e interpretam as demandas paradoxais vindas de seus líderes pode levá-los a vivenciar certo grau de incerteza, conflitos e de percepção de incoerência (Bashir, 2021Bashir, F. (2021). Dark side of paradoxical leadership: social comparison theory perspective. NICE Research Journal, 14(1), 107-130. Recuperado de https://www.nicerjss.com/index.php/JFME/article/download/263/154
https://www.nicerjss.com/index.php/JFME/...
) e, consequentemente, a resistir e reagir de forma defensiva (Schad, Lewis, Raisch, & W. K. Smith, 2016Schad, J., Lewis, M. W., Raisch, S., & Smith, W. K. (2016). Paradox research in management science: looking back to move forward. Academy of Management Annals, 10(1), 5-64. Recuperado de https://doi.org/10.5465/19416520.2016.1162422
https://doi.org/10.5465/19416520.2016.11...
).

Diante disso, um elemento essencial para a literatura de liderança paradoxal consiste em examinar as dinâmicas interacionais relativas ao relacionamento entre líderes e seguidores, em termos de como ambos respondem a demandas paradoxais (Sparr et al., 2022Sparr, J. L., Knippenberg, D. van, & Kearney, E. (2022). Paradoxical leadership as sensegiving: stimulating change-readiness and change-oriented performance. Leadership & Organization Development Journal, 43(2), 225-237. Recuperado de https://doi.org/10.1108/LODJ-04-2021-0161
https://doi.org/10.1108/LODJ-04-2021-016...
; Xue, Li, Liang, & Li, 2020Xue, Y., Li, X., Liang, H., & Li, Y. (2020). How does paradoxical leadership affect employees’ voice behaviors in workplace? A leader-member exchange perspective. International Journal of Environmental Research and Public Health, 17(4), 1162-1185. Recuperado de https://doi.org/10.3390/ijerph17041162
https://doi.org/10.3390/ijerph17041162...
). Há uma visão consensual de que o líder possui um papel central na tarefa de construir uma interação com seus seguidores, na qual ambos possam contribuir para uma resposta coletiva satisfatória a demandas paradoxais (Sparr, 2018DeRue, D. S., Ashford, S. J., & Cotton, N. C. (2009). Assuming the mantle: unpacking the process by which individuals internalize a leader identity. In L. M. Roberts & J. E. Dutton (Eds.), Exploring positive identities and organizations: Building a theoretical and research foundation (pp. 213-232). London, UK: Psychology Press.; Zhang et al., 2015Zhang, Y., Waldman, D. A., Han, Y. L., & Li, X. B. (2015). Paradoxical leader behavior in people management: antecedents and consequences. Academy of Management Journal, 58(2), 538-566. Recuperado de https://doi.org/10.5465/amj.2012.0995
https://doi.org/10.5465/amj.2012.0995...
). Isso justifica que o fulcro da literatura sobre liderança paradoxal seja examinar basicamente os comportamentos do líder, colocando em segundo plano a interação deste com seus seguidores (Park, Shim, Hai, Kwon, & Kim, 2021Park, I. J., Shim, S. H., Hai, S., Kwon, S., & Kim, T. G. (2021). Cool down emotion, don’t be fickle! The role of paradoxical leadership in the relationship between emotional stability and creativity. International Journal of Human Resource Management, 33(14) 2856-2886. Recuperado de https://doi.org/10.1080/09585192.2021.1891115
https://doi.org/10.1080/09585192.2021.18...
). Entretanto, há uma necessidade de melhor compreender a dinâmica interacional entre líderes e seguidores quando os primeiros se definem como paradoxais em seu estilo de liderança (Felix, 2020aFelix, B. (2020a). Analyzing the formation of a paradoxical organizational identity. International Journal of Organizational Analysis, 28(6), 1227-1241. Recuperado de https://doi.org/10.1108/IJOA-08-2019-1849
https://doi.org/10.1108/IJOA-08-2019-184...
; Fiol, 2002Fiol, C. M. (2002). Capitalizing on paradox: the role of language in transforming organizational identities. Organization Science, 13(6), 653-666. Recuperado de https://doi.org/10.1287/orsc.13.6.653.502
https://doi.org/10.1287/orsc.13.6.653.50...
). Neste trabalho, usamos o conceito de identidades para explorar essa dinâmica.

Identidade

De acordo com a Teoria da Identidade Social, de Tajfel e Turner (1979Tajfel, H., & Turner, J. (1979). An integrative theory of intergroup conflict. In Organizational Identity (pp. 56-65). New York, NY: Oxford University Press.), uma identidade é uma autodefinição feita por um indivíduo, construída com base em suas interações sociais, na busca por responder às perguntas “quem sou eu?” ou “quem somos nós?” (Ashforth, Schinoff, & Rogers, 2016Ashforth, B. E., Schinoff, B. S., & Rogers, K. M. (2016). “I identify with her,” “I identify with him”: unpacking the dynamics of personal identification in organizations. Academy of Management Review, 41(1), 28-60. Recuperado de https://doi.org/10.5465/amr.2014.0033
https://doi.org/10.5465/amr.2014.0033...
). Assim, uma identidade de liderança paradoxal ocorre quando um líder se vê como alguém que concilia o alcance de demandas contraditórias e interdependentes ao mesmo tempo. Com base nessa abordagem, adotamos aqui o termo “identidade” de forma intercambiável com “self”, prática esta recorrente na literatura (p. ex., Gomes & Felix, 2019Gomes, R., & Felix, B. (2019). In the closet: a grounded theory of the silence of gays and lesbians in the workplace. Cadernos EBAPE.BR, 17(2), 375-388. Recuperado de https://doi.org/10.1590/1679-395174796
https://doi.org/10.1590/1679-395174796...
; Leavitt & Sluss, 2015Leavitt, K., & Sluss, D. M. (2015). Lying for who we are: an identity-based model of workplace dishonesty. Academy of Management Review, 40(4), 587-610. Recuperado de https://doi.org/10.5465/amr.2013.0167
https://doi.org/10.5465/amr.2013.0167...
). De acordo com Petriglieri (2011Petriglieri, J. L. (2011). Under threat: responses to and the consequences of threats to individuals’ identities. Academy of Management Review, 36(4), 641-662. Recuperado de https://doi.org/10.5465/amr.2009.0087
https://doi.org/10.5465/amr.2009.0087...
), identidades: são múltiplas (um indivíduo não se define de uma única forma, mas de várias); dinâmicas (podem se alterar com o passar do tempo); e levam à criação de autovalor (produzem autoestima).

Desse modo, identidades são múltiplas, na medida em que indivíduos constroem diferentes selves e as ativam levando em consideração características de seus interlocutores, assim como outras questões, constituintes das interações sociais, relacionadas ao contexto (Felix & Cavazotte, 2019Felix, B., & Cavazotte, F. (2019). When a calling goes unanswered: exploring the role of workplace personalizations as calling enactments. Frontiers in Psychology, 10(1940), 1-14. Recuperado de https://doi.org/10.3389/fpsyg.2019.01940
https://doi.org/10.3389/fpsyg.2019.01940...
; Felix, Galon, & Amaro, 2023Felix, B., Fernandes, T., & Mansur, J. (2023). Building (and breaking) a vicious cycle formed by extreme working conditions, work intensification, and perceived well-being: a study of dirty workers in Brazilian favelas. The International Journal of Human Resource Management. Recuperado de https://doi.org/10.1080/09585192.2023.2237864
https://doi.org/10.1080/09585192.2023.22...
; Obodaru, 2017Obodaru, O. (2017). Forgone, but not forgotten: toward a theory of forgone professional identities. Journal: Academy of Management Journal, 60(2), 523-553. Recuperado de https://doi.org/10.5465/amj.2013.0432
https://doi.org/10.5465/amj.2013.0432...
). Com base nessa ideia, entende-se que o fato de alguém adotar uma identidade de liderança paradoxal não significa que outras identidades conflitantes e relacionadas à liderança não possam coexistir.

Identidades podem mudar com o tempo, apesar de fornecerem estabilidade cognitiva e emocional ao autoconceito (Conroy & O’Leary-Kelly, 2014Conroy, S. A., & O’Leary-Kelly, A. M. (2014). Letting go and moving on: work-related identity loss and recovery. Academy of Management Review, 39(1), 67-87. Recuperado de https://doi.org/10.5465/amr.2011.0396
https://doi.org/10.5465/amr.2011.0396...
). O self é um reflexo do contexto social (Mead, 1934Mead, G. H. (1934). Mind, Self and Society. Chicago, IL: The University of Chicago Press.) e, como esse contexto tende a ser alterado ao longo da vida de um indivíduo, mudanças de identidade não são raras. Por consequência, é possível sugerir que, quando um líder constrói uma identidade de liderança paradoxal, não significa que seu self necessariamente se manterá estável e seu estilo de liderança e seu autoconceito não possam retornar futuramente para um self não paradoxal.

Por fim, identidades refletem como pessoas interpretam a si mesmas. Essa interpretação tende a ser feita de forma que leve os indivíduos a possuir uma visão positiva de si mesmos (Dutton, Roberts, & Bednar, 2010Dutton, J. E., Roberts, L. M., & Bednar, J. (2010). Pathways for positive identity construction at work: four types of positive identity and the building of social resources. Academy of Management Review, 35(2), 265-293. Recuperado de https://doi.org/10.5465/amr.35.2.zok265
https://doi.org/10.5465/amr.35.2.zok265...
). Assim, uma das funções de uma identidade é conferir aos indivíduos um nível satisfatório de estabilidade emocional e cognitiva, característica essa que é vista como fundamental para que indivíduos sejam produtivos e encontrem satisfação no trabalho (Felix, Fernandes, & Mansur, 2023Felix, B., Galon, S. Z., & Amaro, R. D. A. (2023). How do women balance multiple roles during the post-maternity-leave period? Community, Work & Family. Recuperado de https://doi.org/10.1080/13668803.2023.2199132
https://doi.org/10.1080/13668803.2023.21...
). Cooper & Thatcher, 2010Cooper, D., & Thatcher, S. M. B. (2010). Identification in organizations: the role of self-concept orientations and identification motives. Academy of Management Review, 35(4), 516-538. Recuperado de https://doi.org/10.5465/amr.35.4.zok516
https://doi.org/10.5465/amr.35.4.zok516...
). Portanto a identidade de liderança paradoxal só será construída caso os indivíduos em questão admirem a noção de gestão de paradoxos, a ponto de terem uma visão positiva de si mesmos ao se verem como portadores de uma mentalidade de paradoxos.

Ameaça à identidade

De forma diferente do que acontece com a maioria dos conceitos dentro do tema de identidades (Felix, Dourado & Nossa, 2023Felix, B., Galon, S. Z., & Amaro, R. D. A. (2023). How do women balance multiple roles during the post-maternity-leave period? Community, Work & Family. Recuperado de https://doi.org/10.1080/13668803.2023.2199132
https://doi.org/10.1080/13668803.2023.21...
), ameaça à identidade não possui uma definição que seja dominantemente aceita na literatura (Petriglieri, 2011Petriglieri, J. L. (2011). Under threat: responses to and the consequences of threats to individuals’ identities. Academy of Management Review, 36(4), 641-662. Recuperado de https://doi.org/10.5465/amr.2009.0087
https://doi.org/10.5465/amr.2009.0087...
). É possível encontrar definições amplas (Elsbach & Kramer, 1996Elsbach, K. D., & Kramer, R. M. (1996). Members’ responses to organizational identity threats: Encountering and countering the Business Week rankings.Administrative Science Quarterly, 41(3), 442-476. Recuperado de https://doi.org/10.2307/2393938
https://doi.org/10.2307/2393938...
; Kreiner & Sheep, 2009Kreiner, G. E., & Sheep, M. L. (2009). Growing pains and gains: Framing identity dynamics as opportunities for identity growth. In L. M. Roberts & J. E. Dutton (Eds.), Exploring positive identities and organizations: Building a theoretical and research foundation (pp. 213-232). London, UK: Psychology Press.), associadas a ambientes pontuais (Elsbach, 2003Elsbach, K. D., & Kramer, R. M. (1996). Members’ responses to organizational identity threats: Encountering and countering the Business Week rankings.Administrative Science Quarterly, 41(3), 442-476. Recuperado de https://doi.org/10.2307/2393938
https://doi.org/10.2307/2393938...
; Major & O’Brien, 2005Major, B., & O’brien, L. T. (2005). The social psychology of stigma. Annual Review of Psychology, 56, 393-421. Recuperado de https://doi.org/10.1146/annurev.psych.56.091103.070137
https://doi.org/10.1146/annurev.psych.56...
) e até estudos que não definem o conceito, apesar de o utilizar (Anteby, 2008Anteby, M. (2008). Identity incentives as an engaging form of control: Revisiting leniencies in an aeronautic plant. Organization Science, 19(2), 202-220. Recuperado de https://doi.org/10.1287/orsc.1070.0343
https://doi.org/10.1287/orsc.1070.0343...
; Ashforth, Kreiner, Clark, & Fugate, 2007Ashforth, B. E., Kreiner, G. E., Clark, M. A., & Fugate, M. 2007. Normalizing dirty work: Managerial tactics for countering occupational taint. Academy of Management Journal, 50(1), 149 -174. Recuperado de https://doi.org/10.5465/amj.2007.24162092
https://doi.org/10.5465/amj.2007.2416209...
). Na literatura, observam-se algumas definições para o termo em estudos nos campos das organizações, psicologia social e sobre estresse.

Neste artigo, optamos por uma definição abrangente e que sintetiza os principais elementos abordados nas demais definições encontradas na literatura. Assim, entendemos ameaças à identidade como “[...] experiências avaliadas como indicativas de dano potencial ao valor, significados e representação de uma identidade” (Petriglieri, 2011Petriglieri, J. L. (2011). Under threat: responses to and the consequences of threats to individuals’ identities. Academy of Management Review, 36(4), 641-662. Recuperado de https://doi.org/10.5465/amr.2009.0087
https://doi.org/10.5465/amr.2009.0087...
, p. 644). Levando em consideração a ideia de que a promoção da autoestima é uma das funções de uma identidade (Dutton et al., 2010Dutton, J. E., Roberts, L. M., & Bednar, J. (2010). Pathways for positive identity construction at work: four types of positive identity and the building of social resources. Academy of Management Review, 35(2), 265-293. Recuperado de https://doi.org/10.5465/amr.35.2.zok265
https://doi.org/10.5465/amr.35.2.zok265...
), é possível dizer que, quando um indivíduo possui um autoconceito que inclui ser um líder paradoxal, ele o faz porque acredita que essa forma de liderança seja positiva. No entanto, quando outras pessoas contestam ou criticam o valor da liderança paradoxal (Felix, 2021Felix, B. (2021). Batalhando com espadas e escudos: uma análise semântica do paradoxo entre pertencimento e desempenho em uma cooperativa. Revista Organizações & Sociedade, 28(99), 786-805. Recuperado de https://doi.org/10.1590/1984-92302021v28n9903PT
https://doi.org/10.1590/1984-92302021v28...
), a depender de como o líder assimile e interprete o fato, isso poderá ser caracterizado como uma ameaça à identidade.

Conquanto as ameaças à identidade sejam frequentemente vistas como originadas na figura de um ameaçador, o lócus da ameaça é sempre o indivíduo que tem sua identidade ameaçada e não o que a originou (Petriglieri, 2011Petriglieri, J. L. (2011). Under threat: responses to and the consequences of threats to individuals’ identities. Academy of Management Review, 36(4), 641-662. Recuperado de https://doi.org/10.5465/amr.2009.0087
https://doi.org/10.5465/amr.2009.0087...
). Uma ameaça à identidade ocorre quando o sujeito que recebe a ação atribui à interação, com outro significante, a interpretação de que algo afetou seu autoconceito (C. A. Smith, 1991Smith, C. A. (1991). The self, appraisal, and coping. In C. R. Snyder & D. R. Forsyth (Eds.), Handbook of social and clinical psychology: the health perspective (pp. 116-137). Elmsford, NY: Pergamon Press.). Por essa razão, é comum o entendimento de que uma mesma situação pode ser vista como positiva ou neutra por uma pessoa, enquanto outra pode vê-la como uma ameaça à identidade (Leavitt & Sluss, 2015Leavitt, K., & Sluss, D. M. (2015). Lying for who we are: an identity-based model of workplace dishonesty. Academy of Management Review, 40(4), 587-610. Recuperado de https://doi.org/10.5465/amr.2013.0167
https://doi.org/10.5465/amr.2013.0167...
).

Como afetam o senso de autoestima dos indivíduos, as ameaças à identidade não costumam ser ignoradas. Não raramente, quando se encontram diante de situações assimiladas como ameaças à identidade, muitas pessoas costumam empregar respostas de enfrentamento para minimizar os efeitos negativos à sua identidade (C. A. Smith, 1991Smith, C. A. (1991). The self, appraisal, and coping. In C. R. Snyder & D. R. Forsyth (Eds.), Handbook of social and clinical psychology: the health perspective (pp. 116-137). Elmsford, NY: Pergamon Press.). Essas respostas de enfrentamento são discutidas principalmente nas literaturas sobre discriminação e estigmatização (Berjot & Gillet, 2011Berjot, S., & Gillet, N. (2011). Stress and coping with discrimination and stigmatization. Frontiers in Psychology, 2(33), 1-13. Recuperado de https://doi.org/10.3389/fpsyg.2011.00033
https://doi.org/10.3389/fpsyg.2011.00033...
; Wehrle, Klehe, Kira, & Zikic, 2018Wehrle, K., Klehe, U. C., Kira, M., & Zikic, J. (2018). Can I come as I am? Refugees’ vocational identity threats, coping, and growth. Journal of Vocational Behavior, 105, 83-101. Recuperado de https://doi.org/10.1016/j.jvb.2017.10.010
https://doi.org/10.1016/j.jvb.2017.10.01...
).

Interpretar eventos como ameaças à identidade leva indivíduos à adoção de respostas antecipatórias de enfrentamento, as quais possuem a função de buscar minimizar os efeitos negativos (Van Laar et al., 2019Van Laar, C., Meeussen, L., Veldman, J., Van Grootel, S., Sterk, N., & Jacobs, C. (2019). Coping with stigma in the workplace: Understanding the role of threat regulation, supportive factors, and potential hidden costs. Frontiers in psychology, 10(1879), 1-21. Recuperado de https://doi.org/10.3389/fpsyg.2019.01879
https://doi.org/10.3389/fpsyg.2019.01879...
). Nesse processo, indivíduos comumente tendem a utilizar recursos cognitivos e emocionais para amenizar os danos causados pela ameaça. Isso significa que eles tendem a refletir e chegar a novas conclusões a respeito do evento percebido ou buscar gerenciar as fronteiras de suas emoções para que sua autoestima não venha a ser afetada por uma ameaça à identidade (Major & O’Brien, 2005Major, B., & O’brien, L. T. (2005). The social psychology of stigma. Annual Review of Psychology, 56, 393-421. Recuperado de https://doi.org/10.1146/annurev.psych.56.091103.070137
https://doi.org/10.1146/annurev.psych.56...
). Esses processos cognitivos e emocionais costumam estar direcionados tanto à fonte da ameaça (o indivíduo ou a situação que levou à interpretação de ameaça) como à identidade ameaçada em si (Petriglieri, 2011Petriglieri, J. L. (2011). Under threat: responses to and the consequences of threats to individuals’ identities. Academy of Management Review, 36(4), 641-662. Recuperado de https://doi.org/10.5465/amr.2009.0087
https://doi.org/10.5465/amr.2009.0087...
). Um exemplo de processo cognitivo em uma resposta de enfrentamento a uma ameaça à identidade seria a reflexão de um indivíduo a respeito de quanto o contexto pode tê-lo influenciado a adotar uma certa ação que levou outra pessoa a contestar determinado aspecto de sua identidade. Já os processos emocionais podem ser exemplificados por uma situação na qual o indivíduo, ao interpretar que sua identidade foi ameaçada, sente raiva do interagente de quem partiu a ameaça original e passa a desacreditá-la por entender que o indivíduo responsável por originá-la não está em uma posição que lhe permita julgá-lo.

Após a revisão da literatura relativa aos conceitos e teorias que fundamentam este estudo, apresentamos na seção seguinte os aspectos metodológicos empregados.

CARACTERIZAÇÃO DO CASO

CoopFin (nome fictício), a instituição onde trabalham os líderes entrevistados neste estudo, é uma das principais instituições financeiras do Brasil. Ela é constituída por um grupo de cooperativas de crédito que oferecem ao mercado serviços geralmente fornecidos por bancos, como empréstimos, contas bancárias, cartões de crédito, consórcios e investimentos. Apesar de sua similaridade com bancos, a CoopFin é uma cooperativa, o que significa dizer que seus clientes são considerados proprietários da organização e que os lucros (chamados de “sobras”) obtidos ao fim de um exercício é distribuído para seus associados. Essa cooperativa foi fundada em 1997, com um propósito de prover crédito rural em regiões menos favorecidas do Brasil. Isso revela sua vocação social e/ou idealista. Assim, com as vantagens fiscais de ser uma cooperativa de crédito, e não um banco, a CoopFin se caracterizou por viabilizar à sociedade serviços bancários com condições mais favoráveis ao consumidor do que aquelas oferecidas por bancos. No momento da realização deste estudo, a empresa possuía 898 funcionários em posição de liderança.

No entanto, seu caminho não foi trilhado sem dificuldades. Em 2006, ano que revelou uma crise no sistema de cooperativas brasileiro, quando ocorreu a falência de várias cooperativas, a CoopFin foi fortemente pressionada pelo Banco Central do Brasil a demonstrar uma sólida consistência e estabilidade financeira para que pudesse seguir com suas operações. Em resposta a tais pressões, a cooperativa passou a implementar uma gestão mais baseada na busca por desempenho financeiro. Diversos novos gerentes que trabalhavam no sistema bancário tradicional foram contratados para facilitar essa mudança de orientação estratégica. O resultado foi o crescimento da cooperativa em áreas urbanas do país, bem como um crescimento substancial no capital social da instituição.

Tal movimento, entretanto, gerou um efeito colateral: diversos bancos se manifestaram ao Banco Central, queixando-se de que a CoopFin desfrutava dos benefícios fiscais de ser uma cooperativa, mas atuava, de fato, com práticas de mercado típicas de um banco. Pressionada, a organização passou em 2011 por um movimento de resgate da cultura cooperativista, mediante redução de suas margens e da condução de um treinamento intensivo de seus líderes para que incorporassem os valores cooperativistas em suas práticas de gestão.

Em 2018, após uma nova crise financeira causada por certo comodismo gerado pelo movimento anterior em prol dos valores cooperativistas, a presidência da cooperativa iniciou um programa de treinamento em liderança paradoxal. A ideia era de que a nova orientação estratégica da CoopFin fosse suportada por uma mudança no comportamento de seus líderes: em vez de enfatizar os valores do cooperativismo (idealismo) ou a busca por resultados financeiros (pragmatismo), os líderes deveriam enfatizar ambos simultaneamente. Dessa forma, tais líderes estariam adotando uma mentalidade de paradoxo, em que ambas as polaridades seriam vistas como interdependentes, a despeito da tensão que guardam entre si. Nesse programa, os líderes foram incentivados a buscar meios de alcançar resultados financeiros via valores cooperativistas, e não apesar deles. Esse mesmo entendimento de que a dualidade “valores cooperativistas” e “metas financeiras” poderia ser interpretada como um paradoxo é encontrado em estudos anteriores sobre instituições de microcrédito (Battilana & Dorado, 2010Battilana, J., & Dorado, S. (2010). Building sustainable hybrid organizations: The case of commercial microfinance organizations. Academy of management Journal, 53(6), 1419-1440. Recuperado de https://doi.org/10.5465/amj.2010.57318391
https://doi.org/10.5465/amj.2010.5731839...
) e, mais especificamente, sobre cooperativas de crédito (Felix, 2021Felix, B. (2021). Batalhando com espadas e escudos: uma análise semântica do paradoxo entre pertencimento e desempenho em uma cooperativa. Revista Organizações & Sociedade, 28(99), 786-805. Recuperado de https://doi.org/10.1590/1984-92302021v28n9903PT
https://doi.org/10.1590/1984-92302021v28...
, 2020bFelix, B., & Cavazotte, F. (2019). When a calling goes unanswered: exploring the role of workplace personalizations as calling enactments. Frontiers in Psychology, 10(1940), 1-14. Recuperado de https://doi.org/10.3389/fpsyg.2019.01940
https://doi.org/10.3389/fpsyg.2019.01940...
; Nelson et al., 2016Nelson, T., Nelson, D., Huybrechts, B., Dufays, F., O’Shea, N., & Trasciani, G. (2016). Emergent identity formation and the co-operative: theory building in relation to alternative organizational forms. Entrepreneurship & regional development, 28(3-4), 286-309. Recuperado de https://doi.org/10.1080/08985626.2016.1155744
https://doi.org/10.1080/08985626.2016.11...
). Os mencionados estudos não evidenciam que a relação entre “valores cooperativistas” e “metas financeiras” deva ser tratada como paradoxo, mas, sim, que a adoção dessa mentalidade pode ser aplicada ao caso. Até a data da realização deste estudo, a CoopFin seguia realizando treinamentos com seus líderes a respeito de como exercer uma liderança paradoxal.

O conteúdo do treinamento envolvia uma explicação sobre o conceito de paradoxos com exemplos recorrentes na literatura (performance x pertencimento; organização x aprendizagem, por exemplo), fundamentados em casos práticos. Em seguida, os participantes do treinamento foram incentivados a descrever os paradoxos centrais da CoopFin e como achavam que as polaridades deveriam ser gerenciadas: como uma escolha ou como interdependentes. Nessa etapa, o paradoxo entre idealismo (valores cooperativistas) e pragmatismo (metas financeiras) foi apontado como o principal. Por fim, foram feitas análises das narrativas organizacionais que promoviam cada uma das polaridades desse paradoxo e discutidas as implicações disso para a sua liderança.

MÉTODO

Perfil dos entrevistados e técnica de seleção dos participantes

O presente estudo foi realizado tendo como base uma metodologia de estudo de caso (Yin, 2015Yin, R. K. (2015). Estudo de Caso: Planejamento e métodos. Porto Alegre, RS: Bookman editora.). Foram coletados dados por meio de entrevistas com líderes da CoopFin. Seus líderes receberam, entre dezembro de 2018 e maio de 2019, um treinamento de 6 meses sobre liderança paradoxal, como parte de um programa de redirecionamento estratégico que busca direcionar a organização para o alcance simultâneo de metas financeiras (idealismo) e a promoção dos valores cooperativistas (pragmatismo). Os participantes foram selecionados para o treinamento por terem tido pouco ou nenhum contato com o tema anteriormente. Isso evidencia que o conceito de liderança paradoxal apresentado era parcial ou totalmente novos para eles. A CoopFin possui uma posição de destaque entre as instituições financeiras brasileiras e foi escolhida como o caso objeto deste estudo por ter se envolvido em uma série de mudanças em razão de pressões paradoxais e ter adotado explicitamente um programa de desenvolvimento de liderança paradoxal.

Para selecionar os participantes do estudo, enviamos um e-mail ao Departamento de Recursos Humanos da CoopFin solicitando que ele fosse encaminhado a todos os empregados em posição de liderança e que haviam participado do treinamento sobre liderança iniciado em 2018. Nessa mensagem, apresentamos uma definição de identidade de liderança paradoxal e fizemos um convite para participação em uma pesquisa por meio de entrevista:

Prezado líder da CoopFin,

Recentemente você participou de um treinamento de 6 meses sobre Liderança Paradoxal. Neste treinamento, muito se discutiu sobre o que é um paradoxo, o que é uma liderança paradoxal, e vocês identificaram o paradoxo central da CoopFin: a busca por conciliar o idealismo (valores cooperativistas) com o pragmatismo (resultados financeiros).

Neste momento, em conjunto com um pesquisador parceiro que está realizando um mestrado em Administração, estamos realizando uma pesquisa sobre o tema da identidade da liderança paradoxal. Uma Identidade é uma maneira pela qual um indivíduo se define por meio de suas interações sociais. Por meio de nossas identidades, procuramos responder às perguntas “quem sou eu?” ou “quem somos nós?” Assim, uma identidade de liderança paradoxal ocorre quando um líder se vê como alguém que concilia o alcance de demandas que são contraditórias e interdependentes. No caso da CoopFin, segundo as próprias interpretações de vocês no treinamento, essas demandas seriam entre o idealismo (valores cooperativistas) e o pragmatismo (resultados financeiros).

Agora, nós gostaríamos de perguntar algo a vocês. Com base em tudo o que aprenderam e que escrevemos neste e-mail, você se vê ou já se viu como uma pessoa que tem uma identidade de liderança paradoxal?

Se não, não se preocupe, você pode ignorar esta mensagem. Mas se sim, gostaríamos de convidar você para participar de uma entrevista a respeito do tema. O tempo dela tende a variar, mas é esperado que dure em torno de uma hora. As entrevistas serão feitas no local de trabalho de vocês, de preferência em uma sala onde possamos ter privacidade. Caso você tenha interesse em participar, por favor envie um e-mail para [e-mail de um dos autores] em até 5 dias.

Muito obrigado e sucesso a todos e todas!

Após esse processo, recebemos uma resposta positiva de 89 líderes de diferentes áreas de atuação (gerentes de agência, gerentes comerciais, membros da presidência, diretor de recursos humanos, gerente de tecnologia, gerente de novos produtos e gerente de expansão). Por meio de um processo intercalado de realização de entrevistas e rodadas de análise de dados, adotamos o princípio da amostragem teórica (von Borell de Araujo & Álvaro-Estramiana, 2011von Borell de Araujo, B. F., & Álvaro-Estramiana, J. L. (2011). Ação e estrutura social em Grounded Theory: reflexões sobre uma psicologia social sociológica. Revista Interamericana de Psicologia/Interamerican Journal of Psychology, 45(3), 381-394. Recuperado de https://www.redalyc.org/pdf/284/28425426008.) para selecionar participantes. Isso significa dizer que, em diferentes etapas de coletas de dados, buscamos alterar o perfil dos participantes em termos de idade, sexo, tempo de atuação e como líder na CoopFin, cargo, de forma que o perfil dos participantes possibilitasse uma variabilidade ampla de perspectivas. Assim, o critério para a variação no perfil dos entrevistados a cada etapa de coleta de dados sempre atendeu ao critério de responder a indagações teóricas que nos vinham à mente. Mais especificamente, ao fim da primeira etapa, que contou com 10 entrevistas, buscamos entrevistados com maior experiência de carreira. Ao fim da segunda, na qual realizamos 8 entrevistas, buscamos participantes com menos experiência na CoopFin. Já após a terceira rodada de coleta de dados, que contou com 8 entrevistas, procuramos entrevistar indivíduos em níveis mais estratégicos na hierarquia da empresa. Por fim, após a quarta etapa de construção de dados, na qual entrevistamos 9 indivíduos, buscamos entrevistar mais homens. Na última etapa, 9 novas entrevistas foram realizadas. Fundamentamos todas essas buscas em indagações e suposições teóricas nossas sobre a possibilidade de tais perfis trazerem novas categorias.

As entrevistas foram realizadas até que novas etapas de coleta de dados não representassem mais o surgimento de novas categorias - suficiência teórica - (Charmaz, 2014Charmaz, K. (2014). Constructing grounded theory (2a ed.). London, UK: Saga.). Após a 29ª entrevista, nenhuma nova categoria foi identificada, de acordo com ambos os autores. Assim, foi realizado, entre agosto de 2019 e junho de 2020, um total de 44 entrevistas. Como citado anteriormente, entendemos que o fato de os indivíduos entrevistados terem se identificado como líderes paradoxais seria suficiente para considerar que possuíam uma identidade paradoxal. Caso o intervalo entre o treinamento e a coleta de dados tivesse sido consideravelmente maior, a teoria, a qual sustenta que identidades precisam ser validadas socialmente para serem retidas no autoconceito (DeRue & Ashford, 2010DeRue, D. S., & Ashford, S. J. (2010). Who will lead and who will follow? A social process of leadership identity construction in organizations. Academy of Management Review, 35(4), 627-647. Recuperado de https://doi.org/10.5465/amr.35.4.zok627
https://doi.org/10.5465/amr.35.4.zok627...
), demandaria que tal validação fosse adicionada como um critério para seleção dos participantes. Todavia, como o intervalo entre o término do treinamento e o início das entrevistas foi de apenas 3 meses, consideramos que essa validação ainda não seria possível nesse contexto de transição de identidades. Assim, fundamentando-nos no entendimento de que identidades recém-adquiridas são consideradas como tal com base na definição que indivíduos fazem a seu respeito e, não, ainda, em validação social (Ashforth, 2000Ashforth, B. E. (2000). Role transitions in organizational life: an identity-based perspective. New York, NY: Routledge.). Consideramos, desse modo, o critério adotado para a seleção dos participantes como coerente com a teoria.

Roteiro de entrevistas

As entrevistas semiestruturadas foram realizadas com base em um roteiro composto por tópicos centrais que deveriam ser abordados de maneira informal, de modo que a entrevista se parecesse menos com uma arguição e mais com um diálogo, consequentemente a naturalidade resultante poderia levar à construção de dados mais esclarecedores (Charmaz, 2014Charmaz, K. (2014). Constructing grounded theory (2a ed.). London, UK: Saga.). Os tópicos foram direcionados a: a) compreender a história profissional e de liderança do indivíduo dentro e fora da CoopFin; b) o entendimento sobre liderança paradoxal; c) explorar se o indivíduo realmente se definia como um(a) líder paradoxal; d) identificar situações em que o entrevistado sentiu sua identidade de líder paradoxal ameaçada; e e) compreender como os entrevistados reagiram a cada uma das situações relatadas. As entrevistas foram realizadas por um dos autores pessoalmente nas salas das agências e na sede regional da organização em questão e duraram, em média, 49 minutos. Em todas as entrevistas, houve as presenças somente de entrevistado e entrevistador. A organização envolvida autorizou a realização dessas entrevistas nesses espaços e em horário comercial.

Perfil dos entrevistados

Dos 44 indivíduos entrevistados, metade (22) é composta por homens e a outra metade (22), por mulheres. Sete (15,9%) dos participantes possuem menos de 30 anos, 14 (31,8%) têm entre 30 e 39 anos, 12 (27,3%) possuem entre 40 e 49 anos e 11 (25%) atingiram os 50 anos ou mais. Há 4 anos ou menos, ocupam a função de liderança 24 (54,5%) entrevistados, isto é, a maioria; apenas 4 (9,1%) estão nessa função há um período de 5 a 9 anos; e outros 16, (36,4%) há 10 anos ou mais. Por fim, em relação ao tempo em que atuam como líderes, independentemente da organização, 13 (29,6%) tornaram-se líderes há 4 anos ou menos, 10 (22,7%), entre 5 e 9 anos e 21 (47,7%) há dez anos ou mais.

Procedimentos de análise de dados

Dentre os entrevistados, 12 autorizaram a gravação e transcrição das entrevistas, enquanto 32 autorizaram apenas que o entrevistador fizesse anotações, o que foi respeitado. As análises foram realizadas imediatamente após o fim de cada etapa de coleta de dados, quando novas entrevistas eram feitas para testar o sistema categorial. A cada rodada, as entrevistas gravadas eram transcritas e, em seguida, analisadas juntamente com as anotações, por um dos autores, daquelas que não foram gravadas. Em cada uma dessas etapas de análise, o outro autor recebia a lista dos códigos e categorizava as evidências nestes ou criava códigos, caso entendesse que o significado de uma determinada evidência não possuía aderência com o nome da categoria. Nos casos em que houve divergência de classificação entre os autores, uma pesquisadora doutora que estuda o tema de identidades foi consultada e expôs sua opinião a ambos os autores, que, por fim, chegaram a um consenso em relação a 96% da categorização realizada. Tal procedimento foi inspirado em Kreiner, Hollensbe, e Sheep (2009Kreiner, G. E., & Sheep, M. L. (2009). Growing pains and gains: Framing identity dynamics as opportunities for identity growth. In L. M. Roberts & J. E. Dutton (Eds.), Exploring positive identities and organizations: Building a theoretical and research foundation (pp. 213-232). London, UK: Psychology Press.). Os dados categorizados encontram-se arquivados e em posse dos autores.

Mais especificamente, a análise de dados ocorreu por meio de um processo inspirado nos procedimentos da grounded theory (Charmaz, 2014Charmaz, K. (2014). Constructing grounded theory (2a ed.). London, UK: Saga.). Dessa forma, toda a codificação foi realizada por meio de uma grade aberta de codificação. Inicialmente foi conduzida uma rodada de análise na qual foram gerados códigos de primeira ordem, sempre iniciados por verbos no gerúndio, com uma função meramente descritiva dos dados. Em seguida, esses termos foram classificados em códigos de segunda ordem, mais abstratos e teóricos que os iniciais, que são as cinco respostas de enfrentamento apresentadas no tópico de resultados do estudo. Por último, esses códigos de segunda ordem foram agrupados em duas dimensões agregadas (respostas de reestruturação e de manutenção da identidade de liderança paradoxal). Cabe ressaltar que um mesmo entrevistado apresentou evidências que foram classificadas em mais de uma categoria, uma vez que houve diferentes reações dos entrevistados a distintas situações consideradas como ameaças à identidade. A seguir, apresentamos em detalhes os resultados encontrados no estudo.

RESULTADOS

Respostas de enfrentamento às ameaças à identidade de liderança paradoxal

Nesta seção, apresentamos as respostas de enfrentamento dos entrevistados. Tais respostas foram dispostas em cinco categorias centrais: “abandonando a identidade”, “alterando o significado da identidade”, “relativizando a relevância da ameaça”, “defendendo a identidade” e “desacreditando ameaçadores”. Enquanto as duas primeiras representam formas pelas quais os indivíduos reestruturaram sua identidade paradoxal, as três últimas representam formas pelas quais eles as preservaram. A seguir, descrevemos cada uma das táticas que emergiram dos dados e apresentamos um exemplo de evidência empírica que as fundamentaram.

Abandonando a identidade

A categoria “abandonando a identidade” foi identificada nos depoimentos de 11 participantes. Nela foram agrupados os relatos nos quais os entrevistados alegaram que, em resposta às ameaças a sua liderança paradoxal, decidiram engajar-se em um processo de saída de identidade. Em tais casos, os entrevistados relataram não terem sustentabilidade na transição identitária para a liderança paradoxal. Foram comuns a essa categoria relatos de que a liderança paradoxal acabou gerando um nível elevado de ambiguidade e incerteza nos liderados. Como consequência, a identidade ou não era compreendida ou não era bem-vista por parte relevante da equipe. Diante disso, manter o autoconceito de liderança paradoxal deixou de ser uma fonte de autoestima para o líder e, portanto, deixou de fazer sentido. O relato a seguir ilustra essa resposta.

Para mim, foi muito complicado ter que lidar com os meus liderados. Eles ficaram confusos, eu sinto isso. Eles estavam acostumados comigo sempre falando do cooperativismo, do lado social, de crédito rural, essas coisas. Aí eu assimilei a ideia de paradoxo, vi que fazia sentido, comecei a me sentir bem sendo assim, gerenciando assim. Só é estranho, porque, quando eles me viram sendo tanto do cooperativismo como do resultado, parece que eles apagaram o meu lado cooperativa. Parece que eu virei o capitalista dos capitalistas. Eu vi que não ia dar, as pessoas polarizam mesmo. Larguei mão desse negócio, achei que acaba gerando mais confusão do que qualquer coisa. Minha equipe voltou a trabalhar melhor quando eu parei de ser essa coisa de paradoxo. (E30, gerente comercial de agência, 58 anos, homem, em posição de liderança há 27 anos).

Esse relato, assim como outros similares, indica que alguns liderados tinham dificuldade de interpretar o líder como alguém que demonstrava uma liderança conciliadora de aspectos que guardam uma elevada tensão e contradição entre si. Alguns entrevistados alegaram que, ao exercer influência orientados por uma das polaridades do paradoxo entre valores cooperativistas (idealismo) e resultado financeiro (pragmatismo), foram interpretados como “incoerentes”, “indecisos”, “confusos” e “inconstantes”. Diante dessa rejeição, alguns preferiram “[...] deixar a teoria para trás e não ser mais assim, porque na prática a teoria é outra” (E20, gerente comercial de agência, 32 anos, homem, em posição de liderança há 4 anos). Assim, o abandono da identidade de liderança paradoxal foi uma das respostas encontradas.

Alterando o significado da identidade

Notamos, em alguns casos, uma percepção de que o abandono da identidade de liderança seria custoso para os indivíduos que se sentissem ameaçados, uma vez que havia ocorrido um investimento da cooperativa no treinamento realizado e um movimento estratégico amplo de busca por uma gestão de paradoxos. Para não se oporem abertamente à ideia de liderança paradoxal, alguns participantes apresentaram uma resposta que codificamos como “alterando o significado da identidade”. Nesse caso, não foi adotado um afastamento explícito da identidade de liderança paradoxal, mas uma busca por construir um novo significado para o que vem a ser liderança paradoxal, distinto daquele aprendido no treinamento proporcionado pelo banco. Dentre os 44 entrevistados, 9 apresentaram evidências que foram inseridas nesta categorização. O relato de uma entrevistada exemplifica essa categoria:

A diretoria treinou a gente no curso de liderança paradoxal, mas os meus liderados, não. Então, quando eu voltei do treinamento, eu treinei a minha equipe, falei como era e passei a ser assim, mais de cooperativa e de desempenho. Por um tempo, tentei me vender desse jeito. Mas o pessoal começou a me ver como gerente de banco, e isso aqui dentro é algo bem negativo, a gente sempre critica os bancos, é cultural. Senti que eu estava destruindo uma imagem de boa gestora que eu vinha criando há 15 anos aqui, de uma gerente que tem orgulho do cooperativismo. Aí eu comecei a mudar, falando que liderança paradoxal era ter pessoas na equipe que eram mais cooperativistas e outras que eram mais do lucro e que eu também poderia escolher qual dos dois combinaria mais comigo. Aí eu falava que tinha escolhido o cooperativismo, mas que isso não me fazia deixar de ser um gestor do paradoxo. [...] Mas eu sei que não é isso. (E44, gerente geral de agência, 41 anos, mulher, em posição de liderança há 9 anos).

Esse trecho da entrevista realizada mostra que a entrevistada, ao ter sua identidade de liderança paradoxal associada a uma traição dos valores idealistas do cooperativismo, ressignificou a liderança paradoxal para a sua equipe. Essa não foi a única entrevistada que o fez: outros participantes também relataram que aproveitaram o fato de seus liderados não terem estado no treinamento oferecido pela empresa em 2018 e realizaram uma gestão de identidade (identity work) ao buscar ressignificar a liderança paradoxal. Um dos entrevistados, por exemplo, afirmou que passou a associar-se a uma ideia de ser um líder que ouve tanto os pragmatistas, como os idealistas na CoopFin - identidade essa que se aproxima muito mais da noção de liderança democrática. Entretanto, nem todas as táticas levaram a uma restruturação da identidade da liderança paradoxal. As três categorias a seguir revelam respostas que possibilitaram aos participantes manter sua identidade de liderança paradoxal.

Relativizando a relevância da ameaça

A terceira categoria foi intitulada “relativizando a relevância da ameaça”. Ela abrange situações nas quais o líder manteve sua identidade e, após um momento inicial em que vivenciou uma ameaça à sua liderança paradoxal, optou por diminuir sua sensibilidade à ameaça por considerá-la menos prejudicial ao seu senso de self. Dessa forma, os 13 entrevistados que tiveram relatos inseridos nesta categoria mostraram que a sensibilidade à ameaça pode ser ajustável, uma vez que eles podem desenvolver um vínculo cognitivo com sua identidade de liderança paradoxal. O relato a seguir exemplifica esse achado.

Eu ouvi que eu estava parecendo RH de banco, que eu tinha ficado mole demais, que ia fazer a cooperativa perder a cultura de performance. Teve de tudo. Parece que algumas pessoas da equipe não viram problema, entenderam, mas outras ficaram meio sem entender, achando que eu não me decidia se era uma coisa ou era outra. Deixei claro pro presidente da cooperativa que isso ia acontecer mesmo, e que a gente não pode ligar tanto para isso. Temos que ser firmes se a gente quer criar uma cultura de paradoxo. Leva tempo para as pessoas entenderem e assimilarem esse jeito de liderar. Então é isso, eu sou uma líder que não só vende, mas também eu vendo certo, na pegada do cooperativismo [...]. Vou continuar assim, claro que eu me chateei, me abalei, duvidei se eu estava certa [...]. Mas é isso mesmo. Vou continuar sendo assim. (E27, gerente de Recursos Humanos, 46 anos, mulher, em posição de liderança há 13 anos).

Conforme pode ser notado no relato apresentado, em diversos casos essa estratégia foi acompanhada de um considerável componente cognitivo, o que significa dizer que a identidade de liderança paradoxal era fortemente sustentada por uma crença de que essa abordagem de liderança seria adequada e/ou satisfatória. Quando a participante menciona: “duvidei se eu estava certa”, demonstra que a contestação quanto à eficácia da liderança paradoxal levou-a a sentir uma ameaça à identidade. Ao relativizar a importância da ameaça, os entrevistados preservaram sua identidade de liderança paradoxal e eliminaram a ameaça, mesmo com a permanência das críticas e contestações.

Defendendo a identidade

Outra resposta apresentada por 8 dos entrevistados fundamentou-se em um processo de convencimento do interagente de que a identidade ameaçada seria, na verdade, tão positiva para o indivíduo e para a organização, que deveria ser mantida. Além de ser preservada, essa tática também revela uma compreensão de que se deveria buscar convencer o liderado a também adotar a perspectiva do paradoxo em seu trabalho. Alguns entrevistados citaram que, quando sentiram sua identidade de liderança paradoxal ameaçada, buscaram “dobrar a aposta” ou “ser mais paradoxal ainda” iniciando um processo de persuasão com seu interagente.

Eu vou ser sincero: eu gostei muito do treinamento, e ele mudou mesmo o meu jeito de liderar. Eu focava só em performance financeira, confesso, e aprendi no treinamento que dá para ter resultado por meio do cooperativismo, que isso é o nosso diferencial estratégico mesmo. Eu realmente fiquei convencido disso. Então quando teve alguém na minha agência que achava que eu estava ficando mole, em vez de ceder ao autoquestionamento que vem no primeiro momento, eu ia lá e ia ensinando, tentando convencer a pessoa, transformar a mentalidade dela. Eu preciso do liderado pensando em paradoxo também. (E42, gerente geral de agência, 34 anos, homem, em posição de liderança há 6 anos).

Esse relato ilustra a resposta de enfrentamento “defendendo a identidade”, visto que o entrevistado em questão não somente não cedeu ao autoquestionamento que ocorreu em um primeiro momento, como também buscou convencer seu interlocutor. Assim, o processo de convencimento daquele que origina a ameaça à identidade mostrou-se um mecanismo eficaz para que diversos líderes intensificassem a busca pela reafirmação de seu autoconceito de liderança paradoxal.

Desacreditando ameaçadores

A última tática, observada no depoimento de 14 entrevistados, também permitiu que líderes entrevistados reafirmassem sua identidade de liderança paradoxal. Essa tática foi codificada como “desacreditando ameaçadores” e se refere a uma busca de argumentos ad hominem que enfraqueçam o potencial poder de uma ameaça, visando afetar a imagem do indivíduo que é fonte da contestação. A lógica dessa tática é de que, ao desacreditar aquele que proferiu alguma crítica que veio a ser vivenciada como uma ameaça à identidade paradoxal, sua reputação é afetada e, por consequência, seu argumento perde força.

O gerente da minha agência, meu superior, não gostou muito dessa ideia de paradoxo e algumas vezes já me criticou, falando que eu não dou tanto resultado, que tenho falado muito de cooperativismo. O problema é que eu estou focando em resultado e em cooperativismo, nos dois, mas ele não enxerga. Acha que eu virei igreja. Quando ele vem querendo tirar a minha moral na frente da minha equipe, eu falo brincando assim: ‘e aí, vai virar faca na caveira também’ [...]. Faca na caveira é o grito de guerra lá no Banco A [nome fictício para o banco em que esse funcionário trabalhava antes de ser contratado pela cooperativa]. O pessoal ri, e com isso eu sinto que eu me defendo e sigo podendo ser um líder de paradoxo mesmo. (E20, gerente comercial, 32 anos, homem, em posição de liderança há 4 anos).

Assim como no relato apresentado, diversos entrevistados declararam terem desacreditado os ameaçadores frequentemente por meio de insinuações irônicas, bem-humoradas e depreciativas. Um dos entrevistados chegou a dizer que “[...] brincando a gente pode falar qualquer coisa, inclusive pode queimar o filme do pessoal que veio do [nome de um banco privado] sem pegar tão mal”. Como é possível notar, essa tática tem um relevante componente social, dado que o processo de desacreditar o violador tem como alvo, frequentemente, “convencer o resto da equipe” de que a identidade de liderança paradoxal do indivíduo ameaçado é positiva e digna de admiração.

Tendo apresentado as respostas de enfrentamento identificadas nas entrevistas realizadas, na próxima seção, procedemos à discussão dos resultados.

DISCUSSÃO

O presente estudo foi realizado com o objetivo de compreender como indivíduos com uma identidade de liderança paradoxal respondem a ameaças a essa identidade. Foram identificadas cinco respostas de enfrentamento: duas delas levam a um abandono da identidade de liderança paradoxal, e outras três permitem que o líder preserve sua identidade. Tal resultado permite fazer avançar a literatura sobre liderança paradoxal, por se tratar de um esforço teórico voltado a explicar como indivíduos reagem a resistências a essa forma de liderança. Na literatura sobre paradoxos organizacionais, não são raros os estudos que analisam como instituições buscam superar resistências à gestão paradoxal (Felix, 2021Felix, B. (2021). Batalhando com espadas e escudos: uma análise semântica do paradoxo entre pertencimento e desempenho em uma cooperativa. Revista Organizações & Sociedade, 28(99), 786-805. Recuperado de https://doi.org/10.1590/1984-92302021v28n9903PT
https://doi.org/10.1590/1984-92302021v28...
, 2020aFelix, B., & Cavazotte, F. (2019). When a calling goes unanswered: exploring the role of workplace personalizations as calling enactments. Frontiers in Psychology, 10(1940), 1-14. Recuperado de https://doi.org/10.3389/fpsyg.2019.01940
https://doi.org/10.3389/fpsyg.2019.01940...
; W. K. Smith & Lewis, 2011Smith, W. K., & Lewis, M. W. (2011). Toward a theory of paradox: a dynamic equilibrium model of organizing. Academy of Management Review, 36(2), 381-403. Recuperado de https://doi.org/10.5465/amr.2009.0223
https://doi.org/10.5465/amr.2009.0223...
). No entanto, em um nível individual de análise, ainda que haja estudos a respeito de como líderes tomam decisões baseadas em paradoxos (W. K. Smith, 2014Smith, W. K. (2014). Dynamic decision making: a model of senior leaders managing strategic paradoxes. Academy of Management Journal, 57(6), 1592-1623. Recuperado de https://doi.org/10.5465/amj.2011.0932
https://doi.org/10.5465/amj.2011.0932...
) e dos efeitos da liderança paradoxal (Li et al., 2020Li, X., Xue, Y., Liang, H., & Yan, D. (2020). The impact of paradoxical leadership on employee voice behavior: a moderated mediation model. Frontiers in Psychology, 11(537756), 1-13. Recuperado de https://doi.org/10.3389/fpsyg.2020.537756
https://doi.org/10.3389/fpsyg.2020.53775...
; Pan, 2021Pan, Z. (2021). Paradoxical leadership and organizational citizenship behavior: the serial mediating effect of a paradoxical mindset and personal service orientation. Leadership and Organization Development Journal, 42(6), 869-881. Recuperado de https://doi.org/10.1108/LODJ-08-2020-0351
https://doi.org/10.1108/LODJ-08-2020-035...
), constata-se uma necessidade de melhor compreender como líderes respondem a resistências e ameaças à liderança paradoxal.

Baseados em estudos anteriores sobre ameaças a identidades (Leavitt & Sluss, 2015Leavitt, K., & Sluss, D. M. (2015). Lying for who we are: an identity-based model of workplace dishonesty. Academy of Management Review, 40(4), 587-610. Recuperado de https://doi.org/10.5465/amr.2013.0167
https://doi.org/10.5465/amr.2013.0167...
) e nos achados deste estudo, ressalvamos que as respostas de enfrentamento a ameaças à liderança paradoxal aqui apresentadas não são plenamente racionais e planejadas, como alguns poderiam sugerir. Isso torna o mapeamento realizado especialmente relevante, uma vez que os achados permitem compreender como uma identidade de liderança paradoxal tende a ser mantida ou abandonada diante da vivência de uma ameaça. Esse achado permite uma visão do fenômeno de transição de identidades, que ocorre quando líderes passam a se ver ou deixam de se ver como líderes paradoxais.

Este estudo também possibilita um avanço para a literatura de liderança paradoxal ao adotar uma perspectiva baseada em identidade em relação a esse fenômeno. Até aqui, estudos anteriores vinham explorando o conceito de comportamentos de liderança paradoxal (Zhang et al., 2015Zhang, Y., Waldman, D. A., Han, Y. L., & Li, X. B. (2015). Paradoxical leader behavior in people management: antecedents and consequences. Academy of Management Journal, 58(2), 538-566. Recuperado de https://doi.org/10.5465/amj.2012.0995
https://doi.org/10.5465/amj.2012.0995...
) e o de liderança paradoxal como um estilo de influência sobre seguidores, fundamentado em uma ideia de gestão de polaridades contraditórias, mas interdependentes (Volk et al., 2022Volk, S., Waldman, D. A., & Barnes, C. M. (2022). A circadian theory of paradoxical leadership. Academy of Management Review, 48(4), 611-638. Recuperado de https://doi.org/10.5465/amr.2020.0468
https://doi.org/10.5465/amr.2020.0468...
). Estudos anteriores já exploraram o fenômeno de incorporação, pelos indivíduos, da gestão de paradoxos a suas identidades coletivas (organizacionais) (Felix, 2020aFelix, B. (2020a). Analyzing the formation of a paradoxical organizational identity. International Journal of Organizational Analysis, 28(6), 1227-1241. Recuperado de https://doi.org/10.1108/IJOA-08-2019-1849
https://doi.org/10.1108/IJOA-08-2019-184...
; Fiol, 2002Fiol, C. M. (2002). Capitalizing on paradox: the role of language in transforming organizational identities. Organization Science, 13(6), 653-666. Recuperado de https://doi.org/10.1287/orsc.13.6.653.502
https://doi.org/10.1287/orsc.13.6.653.50...
). Todavia este estudo avança ao discutir o fenômeno de inserção da gestão paradoxal no autoconceito em um nível individual. Assim como estudos anteriores (DeRue & Ashford, 2010DeRue, D. S., & Ashford, S. J. (2010). Who will lead and who will follow? A social process of leadership identity construction in organizations. Academy of Management Review, 35(4), 627-647. Recuperado de https://doi.org/10.5465/amr.35.4.zok627
https://doi.org/10.5465/amr.35.4.zok627...
; DeRue et al., 2009DeRue, D. S., & Ashford, S. J. (2010). Who will lead and who will follow? A social process of leadership identity construction in organizations. Academy of Management Review, 35(4), 627-647. Recuperado de https://doi.org/10.5465/amr.35.4.zok627
https://doi.org/10.5465/amr.35.4.zok627...
), sugerimos que indivíduos podem interiorizar uma identidade como líder ou seguidor, mas acrescentamos que essa internalização pode ir além, ao abranger um conteúdo substantivo (gestão paradoxal) no autoconceito de líder.

É importante também discutir as implicações dos resultados em relação à tensão entre preservar a liderança paradoxal e abandoná-la. Como foi relatado no tópico de métodos, alguns entrevistados expuseram tanto respostas de preservação, como de reestruturação da identidade. Assim, as categorias apresentadas não são vistas como excludentes. De acordo com esse resultado e com a noção de que identidades são múltiplas e não únicas, dentro da circunscrição de diferentes subsistemas sociais, o mesmo indivíduo pode adotar uma estratégia de nutrir sua identidade paradoxal diante de líderes de hierarquia superior e abandoná-la quando diante de seus seguidores. Por um lado, tal resultado contesta a visão estanque e polarizada de parte da literatura sobre identidades, que defende transições como sendo um processo mais linear (p. ex., Petriglieri, 2011Petriglieri, J. L. (2011). Under threat: responses to and the consequences of threats to individuals’ identities. Academy of Management Review, 36(4), 641-662. Recuperado de https://doi.org/10.5465/amr.2009.0087
https://doi.org/10.5465/amr.2009.0087...
). Por outro lado, alinha-se a outros resultados emergentes nesta literatura, os quais propõem que a construção identitária pode envolver movimentos pendulares de experimentação e de coexistência harmônicas de identidades aparentemente conflitantes (p. ex., Hennekam & Ladge, 2023Hennekam, S., & Ladge, J. J. (2023). Free to Be Me? Evolving Gender Expression and the Dynamic Interplay between Authenticity and the Desire to Be Accepted at Work.Academy of Management Journal, 66(5), 1529-1553. Recuperado de https://doi.org/10.5465/amj.2020.1308
https://doi.org/10.5465/amj.2020.1308...
).

CONSIDERAÇÕES FINAIS

Este estudo possibilitou mapear algumas respostas de enfrentamento que indivíduos adotam quando sentem que sua identidade como líder paradoxal é ameaçada. Tal fenômeno é relativamente comum, uma vez que muitas pessoas possuem uma mentalidade de dilema e, portanto, tendem a enxergar a mentalidade paradoxal como confusa ou como um pensamento que revela indecisão (Miron-Spektor et al., 2018Miron-Spektor, E., Ingram, A., Keller, J., Smith, W. K., & Lewis, M. W. (2018). Microfoundations of organizational paradox: the problem is how we think about the problem. Academy of Management Journal, 61(1), 26-45. Recuperado de https://doi.org/10.5465/amj.2016.0594
https://doi.org/10.5465/amj.2016.0594...
). As respostas encontradas nos dados classificam-se em dois grupos: aquelas em que a identidade de liderança paradoxal é preservada e aquelas em que ela é abandonada. Dado que o processo de responder a ameaças à identidade tende a apresentar limitados graus de planejamento e intencionalidade (Petriglieri & Stein, 2012Petriglieri, G., & Stein, M. (2012). The unwanted self: projective identification in leaders’ identity work. Organization Studies, 33(9), 1217-1235. Recuperado de https://doi.org/10.1177/0170840612448158
https://doi.org/10.1177/0170840612448158...
), nossos achados demonstram relevância ao possibilitar que líderes estejam mais conscientes de como reagem a resistências a sua liderança paradoxal.

Este estudo não ocorreu sem limitações, mas nos permite o apontamento de sugestões para pesquisas futuras. Primeiro, conquanto tenha discutido como líderes respondem a ameaças a sua identidade de liderança paradoxal, não exploramos o processo pelo qual um determinado evento, real ou imaginado, caracteriza-se como uma ameaça à identidade estudada. Estudos futuros podem explorar esse fenômeno, por exemplo, analisando a influência da sensibilidade à ameaça (threat sensibility) do líder no ato de interpretar interações como possivelmente danosas ao seu autoconceito. Segundo, este estudo também se limitou a uma descrição das respostas de enfrentamento e não explorou as condições que tornam a ocorrência de cada uma dessas estratégias mais provável. Terceiro, o estudo foi feito em uma organização em que havia um paradoxo entre idealismo (valores cooperativistas) e pragmatismo (desempenho financeiro). Casos que envolvam outros paradoxos poderiam ser explorados e conduzir a outras estratégias de enfrentamento. Quarto, nossa abordagem micro, apesar de propiciar um entendimento relevante sobre o comportamento individual, pode correr o risco de ser descontextualizada. Uma vez que o comportamento individual não ocorre em um vácuo (Felix, Mello, & von Borell, 2018Felix, B., Mello, A., & von Borell, D. (2018). Voices unspoken? Understanding how gay employees co-construct a climate of voice/silence in organisations.The International Journal of Human Resource Management,29(5), 805-828. Recuperado de https://doi.org/10.1080/09585192.2016.1255987
https://doi.org/10.1080/09585192.2016.12...
), seria interessante analisar como o fenômeno das respostas a ameaças à identidade de liderança paradoxal conecta-se com esferas coletivas do comportamento organizacional, como a cultura ou a identidade organizacional. Quinto, apenas entrevistamos os líderes, não coletamos dados a respeito das reverberações dessas respostas de enfrentamento por seus seguidores. Assim, faz-se necessário explorar a experiência dos liderados nesse processo de reestruturação ou preservação da identidade paradoxal.

Os resultados do presente estudo também possibilitam a proposição de sugestões para a prática. A ocorrência de ameaças à identidade de liderança paradoxal, no caso estudado, relacionou-se com o fato de que o treinamento sobre esse tipo de liderança foi ministrado apenas aos líderes. No entanto, a liderança, para ser bem-sucedida, precisa da adesão comportamental dos seguidores, e a interação com os liderados ficou exclusivamente sob a responsabilidade dos líderes. Assim, sugerimos que treinamentos similares sejam realizados não apenas com líderes, mas também com seus seguidores, de forma a diminuir a distância cognitiva em relação à ideia de paradoxos e facilitar a criação de uma cultura ou identidade paradoxal (Felix, 2021Felix, B. (2021). Batalhando com espadas e escudos: uma análise semântica do paradoxo entre pertencimento e desempenho em uma cooperativa. Revista Organizações & Sociedade, 28(99), 786-805. Recuperado de https://doi.org/10.1590/1984-92302021v28n9903PT
https://doi.org/10.1590/1984-92302021v28...
, 2020aFelix, B., & Cavazotte, F. (2019). When a calling goes unanswered: exploring the role of workplace personalizations as calling enactments. Frontiers in Psychology, 10(1940), 1-14. Recuperado de https://doi.org/10.3389/fpsyg.2019.01940
https://doi.org/10.3389/fpsyg.2019.01940...
). Mesmo quando realizados apenas com líderes, seria interessante incluir em tais treinamentos um tópico mostrando as dificuldades de se validar socialmente uma liderança paradoxal, dado que tal abordagem pode levar pessoas a conflitos cognitivos e emocionais diante de uma forma não familiar de lidar com demandas contraditórias (K. Smith & Berg, 1987Smith, K., & Berg, D. (1987). Paradoxes of group life. San Francisco, CA: Josey-Bass. ; Vince & Broussine, 1996Vince, R., & Broussine, M. (1996). Paradox, defense and attachment: Accessing and working with emotions and relations underlying organizational change. Organization Studies, 17(1) 1-21. Recuperado de https://doi.org/10.1177/017084069601700101
https://doi.org/10.1177/0170840696017001...
). É possível também desenvolver estratégias vivenciais de aprendizagem, nas quais os líderes em treinamentos são submetidos a críticas por sua abordagem paradoxal, reagem diante do grupo e recebem feedbacks a respeito do quanto sua resposta de enfrentamento reflete uma visão intencional e estratégica ou apenas intuitiva e reativa. Além disso, a descrição das respostas que, aqui, apresentamos poderá ajudar líderes que estão enfrentando resistência à sua liderança paradoxal a decidir de maneira mais consciente se irão adotar respostas que mantêm ou eliminam sua identidade de liderança paradoxal.

REFERÊNCIAS

  • Albert, S., & Whetten, D. A. (1985). Organizational identity. In L. L. Cummings & B.M. Staw (Eds.), Research in Organizational Behavior (pp. 263-295). London, UK: Jay Press.
  • Anteby, M. (2008). Identity incentives as an engaging form of control: Revisiting leniencies in an aeronautic plant. Organization Science, 19(2), 202-220. Recuperado de https://doi.org/10.1287/orsc.1070.0343
    » https://doi.org/10.1287/orsc.1070.0343
  • Ashforth, B. E. (2000). Role transitions in organizational life: an identity-based perspective New York, NY: Routledge.
  • Ashforth, B. E., Kreiner, G. E., Clark, M. A., & Fugate, M. 2007. Normalizing dirty work: Managerial tactics for countering occupational taint. Academy of Management Journal, 50(1), 149 -174. Recuperado de https://doi.org/10.5465/amj.2007.24162092
    » https://doi.org/10.5465/amj.2007.24162092
  • Ashforth, B. E., & Reingen, P. H. (2014). Functions of dysfunction: managing the dynamics of an organizational duality in a natural food cooperative. Administrative Science Quarterly, 59(3), 474-516. Recuperado de https://doi.org/10.1177/0001839214537811
    » https://doi.org/10.1177/0001839214537811
  • Ashforth, B. E., Schinoff, B. S., & Rogers, K. M. (2016). “I identify with her,” “I identify with him”: unpacking the dynamics of personal identification in organizations. Academy of Management Review, 41(1), 28-60. Recuperado de https://doi.org/10.5465/amr.2014.0033
    » https://doi.org/10.5465/amr.2014.0033
  • Backhaus, L., Reuber, A., Vogel, D., & Vogel, R. (2021). Giving sense about paradoxes: paradoxical leadership in the public sector. Public Management Review, 24(9), 1478-1498. Recuperado de https://doi.org/10.1080/14719037.2021.1906935
    » https://doi.org/10.1080/14719037.2021.1906935
  • Bashir, F. (2021). Dark side of paradoxical leadership: social comparison theory perspective. NICE Research Journal, 14(1), 107-130. Recuperado de https://www.nicerjss.com/index.php/JFME/article/download/263/154
    » https://www.nicerjss.com/index.php/JFME/article/download/263/154
  • Battilana, J., & Dorado, S. (2010). Building sustainable hybrid organizations: The case of commercial microfinance organizations. Academy of management Journal, 53(6), 1419-1440. Recuperado de https://doi.org/10.5465/amj.2010.57318391
    » https://doi.org/10.5465/amj.2010.57318391
  • Berjot, S., & Gillet, N. (2011). Stress and coping with discrimination and stigmatization. Frontiers in Psychology, 2(33), 1-13. Recuperado de https://doi.org/10.3389/fpsyg.2011.00033
    » https://doi.org/10.3389/fpsyg.2011.00033
  • Cavalcanti, A. L., Felix, B., & Mainardes, E. W. (2022). Do tensions lead to positive career satisfaction results? Revista de Administração Mackenzie, 23(3), 1-26. Recuperado de https://doi.org/10.1590/1678-6971/eRAMR220200.en
    » https://doi.org/10.1590/1678-6971/eRAMR220200.en
  • Charmaz, K. (2014). Constructing grounded theory (2a ed.). London, UK: Saga.
  • Conroy, S. A., & O’Leary-Kelly, A. M. (2014). Letting go and moving on: work-related identity loss and recovery. Academy of Management Review, 39(1), 67-87. Recuperado de https://doi.org/10.5465/amr.2011.0396
    » https://doi.org/10.5465/amr.2011.0396
  • Cooper, D., & Thatcher, S. M. B. (2010). Identification in organizations: the role of self-concept orientations and identification motives. Academy of Management Review, 35(4), 516-538. Recuperado de https://doi.org/10.5465/amr.35.4.zok516
    » https://doi.org/10.5465/amr.35.4.zok516
  • Davies, P. G., Spencer, S. J., & Steele, C. M., & (2005). Clearing the air: Identity safety moderates the effects of stereotype threat on women’s leadership aspirations. Journal of Personality and Social Psychology, 88(2), 276-287. Recuperado de https://doi.org/10.1037/0022-3514.88.2.276
    » https://doi.org/10.1037/0022-3514.88.2.276
  • Day, D. v., & Harrison, M. M. (2007). A multilevel, identity-based approach to leadership development. Human Resource Management Review, 17(4), 360-373. Recuperado de https://doi.org/10.1016/j.hrmr.2007.08.007
    » https://doi.org/10.1016/j.hrmr.2007.08.007
  • DeRue, D. S., & Ashford, S. J. (2010). Who will lead and who will follow? A social process of leadership identity construction in organizations. Academy of Management Review, 35(4), 627-647. Recuperado de https://doi.org/10.5465/amr.35.4.zok627
    » https://doi.org/10.5465/amr.35.4.zok627
  • DeRue, D. S., Ashford, S. J., & Cotton, N. C. (2009). Assuming the mantle: unpacking the process by which individuals internalize a leader identity. In L. M. Roberts & J. E. Dutton (Eds.), Exploring positive identities and organizations: Building a theoretical and research foundation (pp. 213-232). London, UK: Psychology Press.
  • Dutton, J. E., Roberts, L. M., & Bednar, J. (2010). Pathways for positive identity construction at work: four types of positive identity and the building of social resources. Academy of Management Review, 35(2), 265-293. Recuperado de https://doi.org/10.5465/amr.35.2.zok265
    » https://doi.org/10.5465/amr.35.2.zok265
  • Eisenhardt, K. M., & Westcott, B. J. (1988). Paradoxical demands and the creation of excellence: The case of just-in-time manufacturing. In R. Quinn & K. Cameron (Eds.), Paradox and transformation: Toward a theory of change in organization and management (pp. 19-54). Cambridge, MA: Ballinger.
  • Elsbach, K. D. (2003). Relating physical environment to self-categorizations: identity threat and affirmation in a non-territorial office space. Administrative Science Quarterly, 48(4), 622-654. Recuperado de https://doi.org/10.2307/3556639
    » https://doi.org/10.2307/3556639
  • Elsbach, K. D., & Kramer, R. M. (1996). Members’ responses to organizational identity threats: Encountering and countering the Business Week rankings.Administrative Science Quarterly, 41(3), 442-476. Recuperado de https://doi.org/10.2307/2393938
    » https://doi.org/10.2307/2393938
  • Felix, B. (2020a). Analyzing the formation of a paradoxical organizational identity. International Journal of Organizational Analysis, 28(6), 1227-1241. Recuperado de https://doi.org/10.1108/IJOA-08-2019-1849
    » https://doi.org/10.1108/IJOA-08-2019-1849
  • Felix, B. (2020b). The (un)speaking self: an identity-based model for employee voice and silence. Cadernos EBAPE.BR, 18(3), 557-571. Recuperado de https://doi.org/10.1590/1679-395120190037
    » https://doi.org/10.1590/1679-395120190037
  • Felix, B. (2021). Batalhando com espadas e escudos: uma análise semântica do paradoxo entre pertencimento e desempenho em uma cooperativa. Revista Organizações & Sociedade, 28(99), 786-805. Recuperado de https://doi.org/10.1590/1984-92302021v28n9903PT
    » https://doi.org/10.1590/1984-92302021v28n9903PT
  • Felix, B., & Cavazotte, F. (2019). When a calling goes unanswered: exploring the role of workplace personalizations as calling enactments. Frontiers in Psychology, 10(1940), 1-14. Recuperado de https://doi.org/10.3389/fpsyg.2019.01940
    » https://doi.org/10.3389/fpsyg.2019.01940
  • Felix, B., Dourado, D., & Nossa, V. (2023). Algorithmic management, preferences for autonomy/security and gig-workers’ wellbeing: A matter of fit? Frontiers in Psychology, 14, 1-12. Recuperado de https://doi.org/10.3389/fpsyg.2023.1088183
    » https://doi.org/10.3389/fpsyg.2023.1088183
  • Felix, B., Fernandes, T., & Mansur, J. (2023). Building (and breaking) a vicious cycle formed by extreme working conditions, work intensification, and perceived well-being: a study of dirty workers in Brazilian favelas. The International Journal of Human Resource Management. Recuperado de https://doi.org/10.1080/09585192.2023.2237864
    » https://doi.org/10.1080/09585192.2023.2237864
  • Felix, B., Galon, S. Z., & Amaro, R. D. A. (2023). How do women balance multiple roles during the post-maternity-leave period? Community, Work & Family Recuperado de https://doi.org/10.1080/13668803.2023.2199132
    » https://doi.org/10.1080/13668803.2023.2199132
  • Felix, B., Mello, A., & von Borell, D. (2018). Voices unspoken? Understanding how gay employees co-construct a climate of voice/silence in organisations.The International Journal of Human Resource Management,29(5), 805-828. Recuperado de https://doi.org/10.1080/09585192.2016.1255987
    » https://doi.org/10.1080/09585192.2016.1255987
  • Fiol, C. M. (2002). Capitalizing on paradox: the role of language in transforming organizational identities. Organization Science, 13(6), 653-666. Recuperado de https://doi.org/10.1287/orsc.13.6.653.502
    » https://doi.org/10.1287/orsc.13.6.653.502
  • Fürstenberg, N., Alfes, K., & Kearney, E. (2021). How and when paradoxical leadership benefits work engagement: the role of goal clarity and work autonomy. Journal of Occupational and Organizational Psychology, 94(3), 672-705. Recuperado de https://doi.org/10.1111/joop.12344
    » https://doi.org/10.1111/joop.12344
  • Gibson, C. B., & Birkinshaw, J. (2004). The antecedents, consequences, and mediating role of organizational ambidexterity. Academy of Management Journal, 47(2), 209-226. Recuperado de https://doi.org/10.5465/20159573
    » https://doi.org/10.5465/20159573
  • Gomes, R., & Felix, B. (2019). In the closet: a grounded theory of the silence of gays and lesbians in the workplace. Cadernos EBAPE.BR, 17(2), 375-388. Recuperado de https://doi.org/10.1590/1679-395174796
    » https://doi.org/10.1590/1679-395174796
  • Hennekam, S., & Ladge, J. J. (2023). Free to Be Me? Evolving Gender Expression and the Dynamic Interplay between Authenticity and the Desire to Be Accepted at Work.Academy of Management Journal, 66(5), 1529-1553. Recuperado de https://doi.org/10.5465/amj.2020.1308
    » https://doi.org/10.5465/amj.2020.1308
  • Howell, J. P., Dorfman, P. W., & Kerr, S. (1986). Moderator variables in leadership research. Academy of Management Review, 11(1), 88-102. Recuperado de https://doi.org/10.5465/amr.1986.4282632
    » https://doi.org/10.5465/amr.1986.4282632
  • Jäger, U. P., & Schröer, A. (2014). Integrated organizational identity: a definition of hybrid organizations and a research agenda. Voluntas, 25(5), 1281-1306. Recuperado de https://doi.org/10.1007/s11266-013-9386-1
    » https://doi.org/10.1007/s11266-013-9386-1
  • Kozica, A. M. F., Gebhardt, C., Müller-Seitz, G., & Kaiser, S. (2015). Organizational identity and paradox: an analysis of the “stable state of instability” of Wikipedia’s identity. Journal of Management Inquiry, 24(2), 186-203. Recuperado de https://doi.org/10.1177/1056492614553275
    » https://doi.org/10.1177/1056492614553275
  • Kreiner, G. E., Hollensbe, E. C., & Sheep, M. L. (2009). Balancing borders and bridges: Negotiating the work-home interface via boundary work tactics. Academy of Management Journal,52(4), 704-730. Recuperado de https://doi.org/10.5465/amj.2009.43669916
    » https://doi.org/10.5465/amj.2009.43669916
  • Kreiner, G. E., Hollensbe, E., Sheep, M. L., Smith, B. R., & Kataria, N. (2015). Elasticity and the dialectic tensions of organizational identity: how can we hold together while we’re pulling apart? Academy of Management Journal, 58(4), 981-1011. Recuperado de https://doi.org/10.5465/amj.2012.0462
    » https://doi.org/10.5465/amj.2012.0462
  • Kreiner, G. E., & Sheep, M. L. (2009). Growing pains and gains: Framing identity dynamics as opportunities for identity growth. In L. M. Roberts & J. E. Dutton (Eds.), Exploring positive identities and organizations: Building a theoretical and research foundation (pp. 213-232). London, UK: Psychology Press.
  • Leavitt, K., & Sluss, D. M. (2015). Lying for who we are: an identity-based model of workplace dishonesty. Academy of Management Review, 40(4), 587-610. Recuperado de https://doi.org/10.5465/amr.2013.0167
    » https://doi.org/10.5465/amr.2013.0167
  • Lewis, M. 2000. Exploring paradox: Toward a more comprehensive guide. Academy of Management Review, 25(4), 760-776. Recuperado de https://doi.org/10.5465/amr.2000.3707712
    » https://doi.org/10.5465/amr.2000.3707712
  • Li, X., Xue, Y., Liang, H., & Yan, D. (2020). The impact of paradoxical leadership on employee voice behavior: a moderated mediation model. Frontiers in Psychology, 11(537756), 1-13. Recuperado de https://doi.org/10.3389/fpsyg.2020.537756
    » https://doi.org/10.3389/fpsyg.2020.537756
  • Lüscher, L. S., & Lewis, M. W. (2008). Organizational change and managerial sensemaking: working through paradox. Academy of Management Journal, 51(2), 221-240. Recuperado de https://doi.org/10.5465/amj.2008.31767217
    » https://doi.org/10.5465/amj.2008.31767217
  • Luthans, F., & Stewart, T. I. (1977). A General Contingency Theory of Management. Academy of Management Review, 2(2), 181-195. Recuperado de https://doi.org/10.5465/amr.1977.4409038
    » https://doi.org/10.5465/amr.1977.4409038
  • Major, B., & O’brien, L. T. (2005). The social psychology of stigma. Annual Review of Psychology, 56, 393-421. Recuperado de https://doi.org/10.1146/annurev.psych.56.091103.070137
    » https://doi.org/10.1146/annurev.psych.56.091103.070137
  • Mead, G. H. (1934). Mind, Self and Society Chicago, IL: The University of Chicago Press.
  • Miron-Spektor, E., Ingram, A., Keller, J., Smith, W. K., & Lewis, M. W. (2018). Microfoundations of organizational paradox: the problem is how we think about the problem. Academy of Management Journal, 61(1), 26-45. Recuperado de https://doi.org/10.5465/amj.2016.0594
    » https://doi.org/10.5465/amj.2016.0594
  • Nelson, T., Nelson, D., Huybrechts, B., Dufays, F., O’Shea, N., & Trasciani, G. (2016). Emergent identity formation and the co-operative: theory building in relation to alternative organizational forms. Entrepreneurship & regional development, 28(3-4), 286-309. Recuperado de https://doi.org/10.1080/08985626.2016.1155744
    » https://doi.org/10.1080/08985626.2016.1155744
  • Obodaru, O. (2017). Forgone, but not forgotten: toward a theory of forgone professional identities. Journal: Academy of Management Journal, 60(2), 523-553. Recuperado de https://doi.org/10.5465/amj.2013.0432
    » https://doi.org/10.5465/amj.2013.0432
  • O’Reilly, C. A., & Tushman, M. L. (2008). Ambidexterity as a dynamic capability: Resolving the innovator’s dilemma. Research in Organizational Behavior, 28, 185-206. Recuperado de https://doi.org/10.1016/j.riob.2008.06.002
    » https://doi.org/10.1016/j.riob.2008.06.002
  • Pan, Z. (2021). Paradoxical leadership and organizational citizenship behavior: the serial mediating effect of a paradoxical mindset and personal service orientation. Leadership and Organization Development Journal, 42(6), 869-881. Recuperado de https://doi.org/10.1108/LODJ-08-2020-0351
    » https://doi.org/10.1108/LODJ-08-2020-0351
  • Park, I. J., Shim, S. H., Hai, S., Kwon, S., & Kim, T. G. (2021). Cool down emotion, don’t be fickle! The role of paradoxical leadership in the relationship between emotional stability and creativity. International Journal of Human Resource Management, 33(14) 2856-2886. Recuperado de https://doi.org/10.1080/09585192.2021.1891115
    » https://doi.org/10.1080/09585192.2021.1891115
  • Pearce, C. L., Wassenaar, C. L., Berson, Y., & Tuval-Mashiach, R. (2019). Toward a theory of meta-paradoxical leadership. Organizational Behavior and Human Decision Processes, 155, 31-41. Recuperado de https://doi.org/10.1016/j.obhdp.2019.03.003
    » https://doi.org/10.1016/j.obhdp.2019.03.003
  • Petriglieri, G., & Stein, M. (2012). The unwanted self: projective identification in leaders’ identity work. Organization Studies, 33(9), 1217-1235. Recuperado de https://doi.org/10.1177/0170840612448158
    » https://doi.org/10.1177/0170840612448158
  • Petriglieri, J. L. (2011). Under threat: responses to and the consequences of threats to individuals’ identities. Academy of Management Review, 36(4), 641-662. Recuperado de https://doi.org/10.5465/amr.2009.0087
    » https://doi.org/10.5465/amr.2009.0087
  • Raisch, S., & Birkinshaw, J. (2008). Organizational ambidexterity: antecedents, outcomes, and moderators. Journal of Management, 34(3), 375-409. Recuperado de https://doi.org/10.1177/0149206308316058
    » https://doi.org/10.1177/0149206308316058
  • Rothenberg, A. (1979). The emerging goddess Chicago: University of Chicago Press.
  • Schad, J., Lewis, M. W., Raisch, S., & Smith, W. K. (2016). Paradox research in management science: looking back to move forward. Academy of Management Annals, 10(1), 5-64. Recuperado de https://doi.org/10.5465/19416520.2016.1162422
    » https://doi.org/10.5465/19416520.2016.1162422
  • Shao, Y., Nijstad, B. A., & Täuber, S. (2019). Creativity under workload pressure and integrative complexity: The double-edged sword of paradoxical leadership. Organizational Behavior and Human Decision Processes, 155, 7-19. Recuperado de https://doi.org/10.1016/j.obhdp.2019.01.008
    » https://doi.org/10.1016/j.obhdp.2019.01.008
  • She, Z., Li, Q., Yang, B., & Yang, B. (2020). Paradoxical leadership and hospitality employees’ service performance: the role of leader identification and need for cognitive closure. International Journal of Hospitality Management, 89, (102524), 1-11. Recuperado de https://doi.org/10.1016/j.ijhm.2020.102524
    » https://doi.org/10.1016/j.ijhm.2020.102524
  • Smith, C. A. (1991). The self, appraisal, and coping. In C. R. Snyder & D. R. Forsyth (Eds.), Handbook of social and clinical psychology: the health perspective (pp. 116-137). Elmsford, NY: Pergamon Press.
  • Smith, K., & Berg, D. (1987). Paradoxes of group life. San Francisco, CA: Josey-Bass.
  • Smith, W. K. (2014). Dynamic decision making: a model of senior leaders managing strategic paradoxes. Academy of Management Journal, 57(6), 1592-1623. Recuperado de https://doi.org/10.5465/amj.2011.0932
    » https://doi.org/10.5465/amj.2011.0932
  • Smith, W. K., Besharov, M. L., Wessels, A. K., & Chertok, M. (2012). A paradoxical leadership model for social entrepreneurs: challenges, leadership skills, and pedagogical tools for managing social and commercial demands. Academy of Management Learning and Education, 11(3), 463-478. Recuperado de https://doi.org/10.5465/amle.2011.0021
    » https://doi.org/10.5465/amle.2011.0021
  • Smith, W. K., & Lewis, M. W. (2011). Toward a theory of paradox: a dynamic equilibrium model of organizing. Academy of Management Review, 36(2), 381-403. Recuperado de https://doi.org/10.5465/amr.2009.0223
    » https://doi.org/10.5465/amr.2009.0223
  • Sousa, M., Cunha, M. P. E., Simpson, A. V., Giustiniano, L., Rego, A., & Clegg, S. (2022). Servus or Pater? How Paradoxical Intent can qualify Leadership: Inductions from the Kingdom of Bhutan. Journal of Change Management: Reframing Leadership and Organizational Practice, 22(3), 321-353. Recuperado de https://doi.org/10.1080/14697017.2022.2032271
    » https://doi.org/10.1080/14697017.2022.2032271
  • Sparr, J. L. (2018). Paradoxes in organizational change: the crucial r ole of leaders’ sensegiving. Journal of Change Management, 18(2), 162-180. Recuperado de https://doi.org/10.1080/14697017.2018. 1446696
    » https://doi.org/10.1080/14697017.2018. 1446696
  • Sparr, J. L., Knippenberg, D. van, & Kearney, E. (2022). Paradoxical leadership as sensegiving: stimulating change-readiness and change-oriented performance. Leadership & Organization Development Journal, 43(2), 225-237. Recuperado de https://doi.org/10.1108/LODJ-04-2021-0161
    » https://doi.org/10.1108/LODJ-04-2021-0161
  • Steele, C. M(1997). A threat in the air: how stereotypes shape intellectual identity and performance. American Psychologist, 52(6), 613-629. Recuperado de https://doi.org/10.1037/0003-066X.52.6.613
    » https://doi.org/10.1037/0003-066X.52.6.613
  • Sulphey, M. M., & Jasim, K. M. (2022). Paradoxical leadership as a moderating factor in the relationship between organizational silence and employee voice: an examination using SEM. Leadership & Organization Development Journal, 43(3), 457-481. Recuperado de https://doi.org/10.1108/LODJ-02-2021-0075
    » https://doi.org/10.1108/LODJ-02-2021-0075
  • Tajfel, H., & Turner, J. (1979). An integrative theory of intergroup conflict. In Organizational Identity (pp. 56-65). New York, NY: Oxford University Press.
  • Trevor, C. O., & Nyberg, A. J. (2008). Keeping your headcount when all about you are losing theirs: downsizing, voluntary turnover rates, and the moderating role of HR practices. Academy of Management Journal, 51(2), 259-276. Recuperado de https://doi.org/10.5465/amj.2008.31767250
    » https://doi.org/10.5465/amj.2008.31767250
  • Van Laar, C., Meeussen, L., Veldman, J., Van Grootel, S., Sterk, N., & Jacobs, C. (2019). Coping with stigma in the workplace: Understanding the role of threat regulation, supportive factors, and potential hidden costs. Frontiers in psychology, 10(1879), 1-21. Recuperado de https://doi.org/10.3389/fpsyg.2019.01879
    » https://doi.org/10.3389/fpsyg.2019.01879
  • Vince, R., & Broussine, M. (1996). Paradox, defense and attachment: Accessing and working with emotions and relations underlying organizational change. Organization Studies, 17(1) 1-21. Recuperado de https://doi.org/10.1177/017084069601700101
    » https://doi.org/10.1177/017084069601700101
  • Volk, S., Waldman, D. A., & Barnes, C. M. (2022). A circadian theory of paradoxical leadership. Academy of Management Review, 48(4), 611-638. Recuperado de https://doi.org/10.5465/amr.2020.0468
    » https://doi.org/10.5465/amr.2020.0468
  • von Borell de Araujo, B. F., & Álvaro-Estramiana, J. L. (2011). Ação e estrutura social em Grounded Theory: reflexões sobre uma psicologia social sociológica. Revista Interamericana de Psicologia/Interamerican Journal of Psychology, 45(3), 381-394. Recuperado de https://www.redalyc.org/pdf/284/28425426008.
  • Waldman, D. A., & Bowen, D. E. (2016). Learning to be a paradox-savvy leader. Academy of Management Perspectives, 30(3), 316-327. Recuperado de https://doi.org/10.5465/amp.2015.0070
    » https://doi.org/10.5465/amp.2015.0070
  • Wehrle, K., Klehe, U. C., Kira, M., & Zikic, J. (2018). Can I come as I am? Refugees’ vocational identity threats, coping, and growth. Journal of Vocational Behavior, 105, 83-101. Recuperado de https://doi.org/10.1016/j.jvb.2017.10.010
    » https://doi.org/10.1016/j.jvb.2017.10.010
  • Xue, Y., Li, X., Liang, H., & Li, Y. (2020). How does paradoxical leadership affect employees’ voice behaviors in workplace? A leader-member exchange perspective. International Journal of Environmental Research and Public Health, 17(4), 1162-1185. Recuperado de https://doi.org/10.3390/ijerph17041162
    » https://doi.org/10.3390/ijerph17041162
  • Yi, L., Mao, H., & Wang, Z. (2019). How paradoxical leadership affects ambidextrous innovation: the role of knowledge sharing. Social Behavior and Personality: An International Journal, 47(4), 1-15. Recuperado de https://doi.org/10.2224/sbp.7636
    » https://doi.org/10.2224/sbp.7636
  • Yin, R. K. (2015). Estudo de Caso: Planejamento e métodos Porto Alegre, RS: Bookman editora.
  • Zhang, Y., Waldman, D. A., Han, Y. L., & Li, X. B. (2015). Paradoxical leader behavior in people management: antecedents and consequences. Academy of Management Journal, 58(2), 538-566. Recuperado de https://doi.org/10.5465/amj.2012.0995
    » https://doi.org/10.5465/amj.2012.0995
  • Zhang, Y., Zhang, Y., Law, K. S., & Zhou, J. (2022). Paradoxical leadership, subjective ambivalence, and employee creativity: effects of employee holistic thinking. Journal of Management Studies, 59(3), 695-723. Recuperado de https://doi.org/10.1111/joms.12792
    » https://doi.org/10.1111/joms.12792

Datas de Publicação

  • Publicação nesta coleção
    08 Jan 2024
  • Data do Fascículo
    Nov-Dec 2023

Histórico

  • Recebido
    26 Jun 2022
  • Aceito
    16 Jun 2023
Fundação Getulio Vargas, Escola Brasileira de Administração Pública e de Empresas Rua Jornalista Orlando Dantas, 30 - sala 107, 22231-010 Rio de Janeiro/RJ Brasil, Tel.: (21) 3083-2731 - Rio de Janeiro - RJ - Brazil
E-mail: cadernosebape@fgv.br